Re: [Nagios-users] check snmp logic

2013-08-29 Thread Jim Avery
On 29 Aug 2013 02:46, Mark Campbell mcc...@psu.edu wrote:

 So I need to utilize some logic in my service checks.

 I have a chiller that if it is not running the discharge temp is not of
 concern.  However if it is running the discharge temp needs to be lower
 than 50 degrees.

 Is there a way to have the check snmp utilize some logic that says if
 running = 0 then ignore discharge temp.  If running = 1 then discharge
 temp must be 50 degrees?

Yes, I did something similar recently where I wanted to check for existence
of an orphaned file only if ftp failed on another server.

The command definition looked like this:

define command {
  command_name check-for-orphan
  command_line   if $USER1$/check_ftp -H $ARG1$ $ARG2$ $USER10$ then
$USER1$/check_multi -s $HOSTADDRESS$ $ARG3$ $USER10$ else exit 0 $USER10$ fi
  register 1
}

Obviously you won't be using check_ftp and check_multi (probably check_snmp
or your own custom plugins instead) but hopefully you can see now how you
can do something similar to run the second check only if the first returns
an ok state (I'm presuming your first check will return an ok state if
running=1 and the second returns ok if temperature  50 degrees).

Where you see $USER10$ here, that refers to a macro in your
/usr/local/nagios/etc/resource.cfg file that gives you a semi-colon like so:

$USER10$=;

It's a bit of a kludge to allow you to have a multi-line command in a
Nagios command definition.

Note that during the time that the first check returns a non-ok state, the
service check will be in an OK state (the exit 0) but will have null
output.  I guess you could put another line with echo something in there
if you want it to say something meaningful.

I hope that makes some sense...

By the way, please don't blame me if this method breaks in some future
version of Nagios - I'm using 3.3.1 on this one.

Cheers,

Jim
--
Learn the latest--Visual Studio 2012, SharePoint 2013, SQL 2012, more!
Discover the easy way to master current and previous Microsoft technologies
and advance your career. Get an incredible 1,500+ hours of step-by-step
tutorial videos with LearnDevNow. Subscribe today and save!
http://pubads.g.doubleclick.net/gampad/clk?id=58040911iu=/4140/ostg.clktrk___
Nagios-users mailing list
Nagios-users@lists.sourceforge.net
https://lists.sourceforge.net/lists/listinfo/nagios-users
::: Please include Nagios version, plugin version (-v) and OS when reporting 
any issue. 
::: Messages without supporting info will risk being sent to /dev/null

Re: [Nagios-users] Nagios check latency

2013-08-13 Thread Jim Avery
On 13 Aug 2013 05:22, Muhamad Faiz muhamad.f...@asia.xchanging.com
wrote:
 My Nagios setup are having high service check latency.



 Check nagios latency result is 240 seconds!
...
 Any advice will be much appreciated.

If you haven't stopped / started the nagios daemon lately, then do that
(/etc/init.d/nagios stop  then  /etc/init.d/nagios start ,  NOT
/etc/init.d/nagios reload).  I find that after a couple of weeks the
service check latency starts creeping up, and stopping/starting the nagios
daemon sorts it.

I hope that helps.

Jim
--
Get 100% visibility into Java/.NET code with AppDynamics Lite!
It's a free troubleshooting tool designed for production.
Get down to code-level detail for bottlenecks, with 2% overhead. 
Download for free and get started troubleshooting in minutes. 
http://pubads.g.doubleclick.net/gampad/clk?id=48897031iu=/4140/ostg.clktrk___
Nagios-users mailing list
Nagios-users@lists.sourceforge.net
https://lists.sourceforge.net/lists/listinfo/nagios-users
::: Please include Nagios version, plugin version (-v) and OS when reporting 
any issue. 
::: Messages without supporting info will risk being sent to /dev/null

Re: [Nagios-users] ESXi/vCenter Monitoring

2013-08-12 Thread Jim Avery
On 12 Aug 2013 16:50, Daniel Ceola daniel.ce...@occfiber.com wrote:

 Hello all,



 I have a lot of host servers as VM’s on our ESXi infrastructure, managed
with vCenter.  We also use vSphere DRS to help keep the load on host
systems balanced.  Due to the DRS, our VM’s can sometimes move around from
being on one host to being on another w/o my knowledge beforehand.  Does
anyone know if there is any sort of plugin available for Nagios Core that
would basically talk to my vCenter system and find out what VM’s are on
which hosts, and automatically update that VM’s Parent directive in the
host definition?

I specify all of the host servers as parents of the VM.  I reckon that
makes perfect sense, and it's dead easy to implement.
--
Get 100% visibility into Java/.NET code with AppDynamics Lite!
It's a free troubleshooting tool designed for production.
Get down to code-level detail for bottlenecks, with 2% overhead. 
Download for free and get started troubleshooting in minutes. 
http://pubads.g.doubleclick.net/gampad/clk?id=48897031iu=/4140/ostg.clktrk___
Nagios-users mailing list
Nagios-users@lists.sourceforge.net
https://lists.sourceforge.net/lists/listinfo/nagios-users
::: Please include Nagios version, plugin version (-v) and OS when reporting 
any issue. 
::: Messages without supporting info will risk being sent to /dev/null

Re: [Nagios-users] ESXi/vCenter Monitoring

2013-08-12 Thread Jim Avery
On 12 Aug 2013 21:49, Daniel Ceola daniel.ce...@occfiber.com wrote:

 I did that, but it made the map a complete mess of lines that I couldn’t
make out.

Yes.  The map doesn't scale well.  I haven't used it for some years now.
--
Get 100% visibility into Java/.NET code with AppDynamics Lite!
It's a free troubleshooting tool designed for production.
Get down to code-level detail for bottlenecks, with 2% overhead. 
Download for free and get started troubleshooting in minutes. 
http://pubads.g.doubleclick.net/gampad/clk?id=48897031iu=/4140/ostg.clktrk___
Nagios-users mailing list
Nagios-users@lists.sourceforge.net
https://lists.sourceforge.net/lists/listinfo/nagios-users
::: Please include Nagios version, plugin version (-v) and OS when reporting 
any issue. 
::: Messages without supporting info will risk being sent to /dev/null

Re: [Nagios-users] how to start nagios with non nagios user

2013-08-02 Thread Jim Avery
On 31 Jul 2013 14:30, , Roy rp...@njit.edu wrote:

 Hi guys,

 I want to start nagios with non root, nagios user. How do I do that.

Something like..

  sudo /etc/init.d/nagios start

should do it.

You'll need to edit /etc/sudoers to allow your non-root user to do this.
Enter 'man sudoers' to read all the gory detail on how to configure it.
However...

If your system is anything like mine (Ubuntu), you'll find you can easily
give users sudo access to run any command as root by adding them to the
'admin' group.  This is by virtue of this entry in /etc/sudoers:

  # Members of the admin group may gain root privileges
  %admin ALL=(ALL) ALL

Although you might decide this gives them too much power...

I hope that helps.

Jim
--
Get your SQL database under version control now!
Version control is standard for application code, but databases havent 
caught up. So what steps can you take to put your SQL databases under 
version control? Why should you start doing it? Read more to find out.
http://pubads.g.doubleclick.net/gampad/clk?id=49501711iu=/4140/ostg.clktrk___
Nagios-users mailing list
Nagios-users@lists.sourceforge.net
https://lists.sourceforge.net/lists/listinfo/nagios-users
::: Please include Nagios version, plugin version (-v) and OS when reporting 
any issue. 
::: Messages without supporting info will risk being sent to /dev/null

Re: [Nagios-users] NRPE Woes Monitoring Windows Servers

2012-01-28 Thread Jim Avery
On 28 January 2012 15:52, Robert Jackson r...@walkermartyn.co.uk wrote:
 Hope someone can point me in the right direction. For any  check_nrpe
 command (except CheckVersion) I get the following:



 CHECK_NRPE: Received 0 bytes from daemon. Check the remote server logs for
 error messages.



 I know this is a bit sketchy

Yes, it is rather.

, but I don’t understand why the CheckVersion
 command would work and anything else won’t. I’m assuming here that because I
 get the version of the NSClient++ returned without error, then NSClient++ is
 configured correctly? I have uncommented the necessary modules within the
 NSC.ini files on the Windows server.

 I’ve tried both CheckMEM and CheckUptime with no success. Can someone please
 help?

Does the nsclient.log file contain anything interesting?

There are umpteen ways to configure NSClient++ and run checks against it.
What specific checks are you trying to run and how are you running them?

--
Try before you buy = See our experts in action!
The most comprehensive online learning library for Microsoft developers
is just $99.99! Visual Studio, SharePoint, SQL - plus HTML5, CSS3, MVC3,
Metro Style Apps, more. Free future releases when you subscribe now!
http://p.sf.net/sfu/learndevnow-dev2
___
Nagios-users mailing list
Nagios-users@lists.sourceforge.net
https://lists.sourceforge.net/lists/listinfo/nagios-users
::: Please include Nagios version, plugin version (-v) and OS when reporting 
any issue. 
::: Messages without supporting info will risk being sent to /dev/null


Re: [Nagios-users] Can passive results cause active check rescheduling?

2012-01-24 Thread Jim Avery
On 24 January 2012 15:07, Craig Stewart craig.stew...@corp.xplornet.com wrote:
 Good day!

 Here is a question.  Can the processing of a passive service check
 schedule a pending active service check?

 Here's a simple example from a distributed monitoring setup.

 Lets say we have a service called check_alive that simply pings a host.
  It's defined on the probe and the central server.  On the probe, it's
 scheduled to check every 60 seconds, and submit the results to the
 central server.  On the central server, it's scheduled to check every
 120 seconds.  Is there a way to cause the following behaviour to occur?

 - Probe checks and submits the results to central.  It reschedules the
 check for 60 seconds from now.

 - Central receives and processes the results (assume an okay state here)
 and reschedules it's active check that was going to run within short
 time frame to 120 seconds from now, without running the active check.

 - 60 seconds later the probe checks and submits (still in an okay state)
 it's check to the central server.

 - The central server receives and processes the check result, and
 reschedules (postpones) it's active service check for 120 seconds from
 now, without actually running it.

 - Repeat until a non-okay result comes in.



 This way if the probe goes silent, the central server will pick up
 monitoring nearly seamlessly. I know I can get the results I want using
 the check freshness option.

 What is happening now, if I enable active service checks on the central
 server is that the passive result comes in, is processed and the active
 service check is run at exactly the scheduled time.  There is no
 postponing the central server's check time.

 There are a couple of problems.

 1) My central server also does active service checking for some devices
 that are not associated with a probe.  I can build these to have a
 separate template, but I'm already getting into template complexity.  I
 don't want to have to have two templates, one for local and one for
 remote monitoring of each device type.

 2) I have an individual that doesn't like to see red on the Nagios
 interface.  They are sufficiently far up the food chain that all I can
 really do is say Yes sir, yes sir, three bags full sir and hope they
 forget about it.  All the 1 plus services show up in the Nagios
 interface as disabled and have a red background.  Please assume any
 valid arguments on my part have been made.

 None of my reading has suggested that this behaviour is possible, but I
 thought I'd put it out there and ask.


My understanding of what Nagios does is it schedules the active check,
it goes in to the schedule queue with the specific time it's scheduled
to run and that is the time it will run regardless of what passive
checks come in in the mean time.

I don't understand why it's a problem to you to have the active checks
run in their regular times under the control of the Nagios scheduler
with passive checks coming in too at whatever times they come in.
Granted you will fairly often have the two checks run at almost the
same time, but does that cause you a problem?

You're right you could use freshness checking.  Is there a particular
reason why you chose not to?

--
Keep Your Developer Skills Current with LearnDevNow!
The most comprehensive online learning library for Microsoft developers
is just $99.99! Visual Studio, SharePoint, SQL - plus HTML5, CSS3, MVC3,
Metro Style Apps, more. Free future releases when you subscribe now!
http://p.sf.net/sfu/learndevnow-d2d
___
Nagios-users mailing list
Nagios-users@lists.sourceforge.net
https://lists.sourceforge.net/lists/listinfo/nagios-users
::: Please include Nagios version, plugin version (-v) and OS when reporting 
any issue. 
::: Messages without supporting info will risk being sent to /dev/null


Re: [Nagios-users] Windows Active Sessions

2012-01-23 Thread Jim Avery
On 23 January 2012 13:09, Robert Jackson r...@walkermartyn.co.uk wrote:
 Does anyone know how to return the number of active sessions from a Windows
 Terminal server? I have installed the NSClient++ service on the Windows
 server and obviously have the NRPE plugin installed on my Nagios host.


Rab,

if I've understood correctly then you can query a Windows performance counter.

I use an entry in the NSClient++ config file in the [External Alias]
section that looks like this:-

  alias_CheckCounter-ts_act_sess=CheckCounter
Counter:sessions=\Terminal Services\Active Sessions ShowAll
MaxWarn=40 MaxCrit=50


Then on the Nagios side, the command is simply check_nrpe -H
thehostname -c alias_CheckCounter-ts_act_sess

So the command definition in Nagios is:-


define command{
  command_namecheck_nrpe
  command_line$USER1$/check_nrpe -H $HOSTADDRESS$ -u $ARG1$
  }


And the service definition looks something like this:-

define service{
  use   srv-pnp,generic-service
  host_name myhost
  service_description   TS_Active_Sessions
  check_command check_nrpe!-c ts_act_sess
  notes Number of Terminal Services Active Sessions
  contact_groupsnotify-engineers
}


There are lots of similar ways to do this sort of think using
NSClient++.  Precisely how you set it up depends a bit on how
obsessive you are about security and your particular preference.

The method shown above means you can have allow_arguments=0 and
allow_nasty_meta_chars=0 in the [nrpe] section which is good for
security.  The downside is that if you need to change the warning
theshold, you have to edit the nsc.ini file on the target system.

hth,

Jim

--
Try before you buy = See our experts in action!
The most comprehensive online learning library for Microsoft developers
is just $99.99! Visual Studio, SharePoint, SQL - plus HTML5, CSS3, MVC3,
Metro Style Apps, more. Free future releases when you subscribe now!
http://p.sf.net/sfu/learndevnow-dev2
___
Nagios-users mailing list
Nagios-users@lists.sourceforge.net
https://lists.sourceforge.net/lists/listinfo/nagios-users
::: Please include Nagios version, plugin version (-v) and OS when reporting 
any issue. 
::: Messages without supporting info will risk being sent to /dev/null


Re: [Nagios-users] Windows Active Sessions

2012-01-23 Thread Jim Avery
On 23 January 2012 14:56, Robert Jackson r...@walkermartyn.co.uk wrote:
 Hi Jim,

 Managed to get some output, but not what I expected:

 I (0.3.8.75 2010-05-27) seem to be doing fine

 I get the same manually running check_nrpe from the Nagios host command
 line.


 Any ideas?

I've not seen that myself, but a quick google for nsclient++ seem to
be doing fine comes up with this.

http://www.nsclient.org/nscp/discussion/topic/510

I think that's very likely your solution.

--
Try before you buy = See our experts in action!
The most comprehensive online learning library for Microsoft developers
is just $99.99! Visual Studio, SharePoint, SQL - plus HTML5, CSS3, MVC3,
Metro Style Apps, more. Free future releases when you subscribe now!
http://p.sf.net/sfu/learndevnow-dev2
___
Nagios-users mailing list
Nagios-users@lists.sourceforge.net
https://lists.sourceforge.net/lists/listinfo/nagios-users
::: Please include Nagios version, plugin version (-v) and OS when reporting 
any issue. 
::: Messages without supporting info will risk being sent to /dev/null


Re: [Nagios-users] Acknowledge link on service commands

2012-01-10 Thread Jim Avery
On 10 January 2012 11:36, Saulo Zimbaro saulo.zimb...@acotel.com wrote:
 I want to know if have a solution for link acknowledge bug in nagios
 services commands. I´ve already upgrade to nagios 3.3.1 but the link still
 not appears for some services.


The Acknowledge button is only available if the service (or host) is
in a Hard state, is not OK and is not already acknowledged.

For information on state types, see:-

http://nagios.sourceforge.net/docs/nagioscore/3/en/statetypes.html

--
Write once. Port to many.
Get the SDK and tools to simplify cross-platform app development. Create 
new or port existing apps to sell to consumers worldwide. Explore the 
Intel AppUpSM program developer opportunity. appdeveloper.intel.com/join
http://p.sf.net/sfu/intel-appdev
___
Nagios-users mailing list
Nagios-users@lists.sourceforge.net
https://lists.sourceforge.net/lists/listinfo/nagios-users
::: Please include Nagios version, plugin version (-v) and OS when reporting 
any issue. 
::: Messages without supporting info will risk being sent to /dev/null


Re: [Nagios-users] Parent/Child relationship for HV/VM

2012-01-04 Thread Jim Avery
On 4 January 2012 07:53, Claudio Kuenzler c...@claudiokuenzler.com wrote:
 I created a fake host (with IP 0.0.0.0) for the cluster with active checks
 and notifications disabled.
 This way I have one and the same parent for all VM's in that cluster.

 Let me show it this way:

 switch --- esx1  VM-CLUSTER-1 -- VM1
 |  esx2  |  VM2
 |  esx3  |  VM3
 |  esx4  |- VM4

 I hope you see what I'm trying to show.
 In text:
 Parent of vm's 1-4 is the 'fake' host VM-CLUSTER-1. Parents of VM-CLUSTER-1
 are esx1, esx2, esx3, esx4. Parent of esx1-4 is switch.

 Positive: You can represent the cluster with a host and in the map it's
 clearly seen which physical hosts are part of this cluster.
 Negative: The host will stay grey (pending) in Nagios as no check will be
 executed on the fake host.

 In Nagvis it looks good and it makes sense.


That's a good idea.

Another advantage is if you add or remove host servers to the cluster
you only need to change the one host definition.

You could use check_cluster as the host check for your fake host, then
its status would be meaningful too.  I'll have a go at doing that
myself when I get a minute.

The documentation for check_cluster if you run check_cluster --help
isn't all that helpful.  Some examples can be found after some
googling for example at:

http://www.nagios-wiki.de/nagios/doku3/clusters

If I recall I learned how to use the check_cluster plugin by reading
Wolfgang Barth's book.

Cheers,

Jim

--
Ridiculously easy VDI. With Citrix VDI-in-a-Box, you don't need a complex
infrastructure or vast IT resources to deliver seamless, secure access to
virtual desktops. With this all-in-one solution, easily deploy virtual 
desktops for less than the cost of PCs and save 60% on VDI infrastructure 
costs. Try it free! http://p.sf.net/sfu/Citrix-VDIinabox
___
Nagios-users mailing list
Nagios-users@lists.sourceforge.net
https://lists.sourceforge.net/lists/listinfo/nagios-users
::: Please include Nagios version, plugin version (-v) and OS when reporting 
any issue. 
::: Messages without supporting info will risk being sent to /dev/null


Re: [Nagios-users] Parent/Child relationship for HV/VM

2012-01-03 Thread Jim Avery
On 3 January 2012 20:58, alexus ale...@gmail.com wrote:
 I have few hyper-visors that running bunch of VMs. Each VM runs on a
 HV and can be migrated from one host to another at any given time for
 whatever reason(s).
 Should I specify all HVs (where VM can run) for each VM as a parent?

That's what I do, yes.

--
Write once. Port to many.
Get the SDK and tools to simplify cross-platform app development. Create 
new or port existing apps to sell to consumers worldwide. Explore the 
Intel AppUpSM program developer opportunity. appdeveloper.intel.com/join
http://p.sf.net/sfu/intel-appdev
___
Nagios-users mailing list
Nagios-users@lists.sourceforge.net
https://lists.sourceforge.net/lists/listinfo/nagios-users
::: Please include Nagios version, plugin version (-v) and OS when reporting 
any issue. 
::: Messages without supporting info will risk being sent to /dev/null


Re: [Nagios-users] freshness checks not working

2011-12-29 Thread Jim Avery
On 27 December 2011 13:32, JM jm+nagios-us...@roth.lu wrote:
 Jim Avery jim at jimavery.me.uk writes:
 Is the the relevant entry for that service check in
 your objects.cache file correct?

 I believe so. Do you see something out of the ordinary:

        host_name       xxx
        service_description     yyy
        check_period    24x7
        check_command   check_dummy!0
        contact_groups  cg_xx
        notification_period     24x7
        initial_state   o
        check_interval  1.00
        retry_interval  1.00
        max_check_attempts      1
        is_volatile     1
        parallelize_check       1
        active_checks_enabled   0
        passive_checks_enabled  1
        obsess_over_service     1
        event_handler_enabled   1
        low_flap_threshold      0.00
        high_flap_threshold     0.00
        flap_detection_enabled  0
        flap_detection_options  o,w,u,c
        freshness_threshold     600
        check_freshness 1
        notification_options    u,w,c
        notifications_enabled   1
        notification_interval   240.00
        first_notification_delay        0.00
        stalking_options        n
        process_perf_data       1
        failure_prediction_enabled      1
        retain_status_information       1
        retain_nonstatus_information    1

Ok looking at my config where I do the same sort of thing to clear
SNMP-Trap states after a period, it's pretty much the same as yours
except I have ..

obsess_over_service 0
event_handler_enabled   0
low_flap_threshold  60.00
high_flap_threshold 80.00
flap_detection_enabled  1
flap_detection_options  o,w,u,c
notification_optionsw,c,r
stalking_optionsw,c
process_perf_data   0
failure_prediction_enabled  0

I'll leave it to you to think about whether any of those are likely to
have any effect.  I don't think they should do, but it might be worth
doing a bit of experimentation.

Cheers,

Jim

--
Ridiculously easy VDI. With Citrix VDI-in-a-Box, you don't need a complex
infrastructure or vast IT resources to deliver seamless, secure access to
virtual desktops. With this all-in-one solution, easily deploy virtual 
desktops for less than the cost of PCs and save 60% on VDI infrastructure 
costs. Try it free! http://p.sf.net/sfu/Citrix-VDIinabox
___
Nagios-users mailing list
Nagios-users@lists.sourceforge.net
https://lists.sourceforge.net/lists/listinfo/nagios-users
::: Please include Nagios version, plugin version (-v) and OS when reporting 
any issue. 
::: Messages without supporting info will risk being sent to /dev/null


Re: [Nagios-users] scheduled downtime nagios.cmd

2011-12-29 Thread Jim Avery
On 29 December 2011 19:23, Jeremiah D. Jester jj...@u.washington.edu wrote:
 I’m attempting to integrate a downtime schedule script I downloaded from
 nagios.org into my installation
 (http://exchange.nagios.org/directory/Addons/Scheduled-Downtime/Downtime-Scheduler/details)
 The script is supposed to write downtime information to the nagios.cmd file
 and disable the referenced host. However, when I tail nagios.cmd I am unable
 to view any processed commands.

The command file isn't a file, it's a pipe so I expect Nagios will
have already read the line by the time you get to attempt to read it,
and if you were able to read it then it wouldn't be there for Nagios
to read it.

 Interestingly enough the command DOES show
 in my nagios.log file so it appears that it is processing the command.



 [1325186243] EXTERNAL COMMAND:
 SCHEDULE_HOST_DOWNTIME;monk;1325186100;1325187540;1;7200;Nagios;Test



 The problem is that I don’t believe the host  (monk) is being disabled
 despite having the start time already occurring in the past. I’ve validated
 this test by disabling nrpe on the client.



 [1325186247] SERVICE ALERT: monk;MEMORY STATUS;CRITICAL;SOFT;1;Connection
 refused by host


Scheduled downtime won't stop Nagios from running active checks or
displaying the resulting alerts in the CGIs.  It should stop Nagios
from sending notifications (emails) though and you should see the
relevant icon to show the host is in a period of scheduled downtime.
To view all scheduled downtimes currently configured in Nagios, click
'downtime' in the 'reports' part of the menu.

--
Ridiculously easy VDI. With Citrix VDI-in-a-Box, you don't need a complex
infrastructure or vast IT resources to deliver seamless, secure access to
virtual desktops. With this all-in-one solution, easily deploy virtual 
desktops for less than the cost of PCs and save 60% on VDI infrastructure 
costs. Try it free! http://p.sf.net/sfu/Citrix-VDIinabox
___
Nagios-users mailing list
Nagios-users@lists.sourceforge.net
https://lists.sourceforge.net/lists/listinfo/nagios-users
::: Please include Nagios version, plugin version (-v) and OS when reporting 
any issue. 
::: Messages without supporting info will risk being sent to /dev/null


Re: [Nagios-users] Newbie: controlling bad packets.

2011-12-28 Thread Jim Avery
On 27 December 2011 14:22, Network Operation Center FMC Luxemburg
n...@eurofmc.com wrote:
 Hi the List,

 I would control if bad packets has been rejected on a given lan for any or
 several machines

 A command like this :

 check_rejected_packets -L lan address, i.e 192.168.0.0, or a lot of
 addresses, or an intervall -T elapsed times in seconds

 Any ideas? A plugin exists for this control?

Take a look at http://nagios.manubulon.com/snmp_int.html

With the -e option, it gives you in/out errors and discards for the
specific interface (if that is the sort of thing you want).

--
Write once. Port to many.
Get the SDK and tools to simplify cross-platform app development. Create 
new or port existing apps to sell to consumers worldwide. Explore the 
Intel AppUpSM program developer opportunity. appdeveloper.intel.com/join
http://p.sf.net/sfu/intel-appdev
___
Nagios-users mailing list
Nagios-users@lists.sourceforge.net
https://lists.sourceforge.net/lists/listinfo/nagios-users
::: Please include Nagios version, plugin version (-v) and OS when reporting 
any issue. 
::: Messages without supporting info will risk being sent to /dev/null


Re: [Nagios-users] freshness checks not working

2011-12-27 Thread Jim Avery
On 27 December 2011 13:32, JM jm+nagios-us...@roth.lu wrote:
 Jim Avery jim at jimavery.me.uk writes:
 Is the the relevant entry for that service check in
 your objects.cache file correct?

 I believe so. Do you see something out of the ordinary:

I can't see anything wrong with that at all, no.

The only difference I can think of between what you have and what I
have is I don't use an event handler.  I'll be back at work on
Thursday so I'll take a closer look then.

--
Write once. Port to many.
Get the SDK and tools to simplify cross-platform app development. Create 
new or port existing apps to sell to consumers worldwide. Explore the 
Intel AppUpSM program developer opportunity. appdeveloper.intel.com/join
http://p.sf.net/sfu/intel-appdev
___
Nagios-users mailing list
Nagios-users@lists.sourceforge.net
https://lists.sourceforge.net/lists/listinfo/nagios-users
::: Please include Nagios version, plugin version (-v) and OS when reporting 
any issue. 
::: Messages without supporting info will risk being sent to /dev/null


Re: [Nagios-users] freshness checks not working

2011-12-23 Thread Jim Avery
On 23 December 2011 18:48,  jm+nagios-us...@roth.lu wrote:
 Hi there,

 consider the following template:

 define service {
 name                           x_trap
 register                       0        ; DONT REGISTER  - ITS NOT
 REAL,JUST A TEMPLATE!
 host_name                      host
 active_checks_enabled          0
 passive_checks_enabled         1
 is_volatile                    1
 max_check_attempts             1
 normal_check_interval          1
 retry_check_interval           1
 flap_detection_enabled         0
 contact_groups                 cg_x
 notification_options           w,u,c
 notification_interval          240
 notification_period            24x7
 check_freshness 1
 freshness_threshold 600
 check_command check_dummy!0
 check_period 24x7
 }

 Can someone tell me why some real services derived from the above template
 do not seem to do the freshness checks?

Are you sure they don't do the freshness check?  Personally I would
have the following arguments (or similar) to check_dummy, using 3
rather than 0 to show an unknown rather than ok state.

  check_command check_dummy!3 UNKNOWN: Nagios did not receive a
passive service check recently.

One thing that catches me out frequently is when I put an explanation
mark in there which stops it from working.  For example, this will not
work.

  check_command check_dummy!3 UNKNOWN: Nagios did not receive a
passive service check recently!

hth,

Jim

--
Write once. Port to many.
Get the SDK and tools to simplify cross-platform app development. Create 
new or port existing apps to sell to consumers worldwide. Explore the 
Intel AppUpSM program developer opportunity. appdeveloper.intel.com/join
http://p.sf.net/sfu/intel-appdev
___
Nagios-users mailing list
Nagios-users@lists.sourceforge.net
https://lists.sourceforge.net/lists/listinfo/nagios-users
::: Please include Nagios version, plugin version (-v) and OS when reporting 
any issue. 
::: Messages without supporting info will risk being sent to /dev/null


Re: [Nagios-users] Feeding status information to SCOM?

2011-12-20 Thread Jim Avery
On 20 December 2011 13:58, Troels Arvin tro...@arvin.dk wrote:

 Thoughts/comments?

In the other direction, we have an IBM Director system which simply
forwards alerts to Nagios as SNMP Traps.  It seems to work pretty well
on the whole.

I would think you could configure your Nagios system to send traps for
any state changes or events to the SCOM system quite easily.  A bit of
googling around will find you the bits and pieces you need to do that,
including the MIB.  For example, I found a FAQ for it here:-
http://support.nagios.com/knowledgebase/faqs/index.php?option=com_contentview=articleid=52catid=35faq_id=28expand=falseshowdesc=true

That's my 2p worth anyway ...

Cheers,

Jim

--
Write once. Port to many.
Get the SDK and tools to simplify cross-platform app development. Create 
new or port existing apps to sell to consumers worldwide. Explore the 
Intel AppUpSM program developer opportunity. appdeveloper.intel.com/join
http://p.sf.net/sfu/intel-appdev
___
Nagios-users mailing list
Nagios-users@lists.sourceforge.net
https://lists.sourceforge.net/lists/listinfo/nagios-users
::: Please include Nagios version, plugin version (-v) and OS when reporting 
any issue. 
::: Messages without supporting info will risk being sent to /dev/null


Re: [Nagios-users] permissions issues with nagiosql

2011-12-13 Thread Jim Avery
On 13 December 2011 16:11, Jonathan Aquilina eagles051...@gmail.com wrote:
 hey guys im having permissions issues and getting the following message

 Cannot open/overwrite the configuration file (check the permissions)!

 i did some googling and game up with
 http://www.nagiosql.org/forum/installation-issues/195-permissions.html

 should i have some template config files somehwere? From what I am
 seeing i do not have any demo configs showing up anywhere

On my system, the user www-data (the apache web server user) is
added to the nagios group.  So, in the /etc/group file we have:-

nagios:x:1001:www-data

If www-data (or whichever user your Apache daemon runs as) isn't in
the 'nagios' group, you should be able to add it something like so ..

  usermod -G nagios www-data

hth,

Jim

--
Systems Optimization Self Assessment
Improve efficiency and utilization of IT resources. Drive out cost and 
improve service delivery. Take 5 minutes to use this Systems Optimization 
Self Assessment. http://www.accelacomm.com/jaw/sdnl/114/51450054/
___
Nagios-users mailing list
Nagios-users@lists.sourceforge.net
https://lists.sourceforge.net/lists/listinfo/nagios-users
::: Please include Nagios version, plugin version (-v) and OS when reporting 
any issue. 
::: Messages without supporting info will risk being sent to /dev/null


Re: [Nagios-users] How to monitor memory on cisco router

2011-12-13 Thread Jim Avery
On 13 December 2011 16:41, moses neah mssn...@yahoo.com wrote:
 Hi all,
 Where can I find a plugin to monitor cisco routers memory. I got the one
 that monitors CPU, fan and temperature.


I haven't tried it myself, but this one should do the trick:-

http://nagios.manubulon.com/snmp_mem.html

--
Systems Optimization Self Assessment
Improve efficiency and utilization of IT resources. Drive out cost and 
improve service delivery. Take 5 minutes to use this Systems Optimization 
Self Assessment. http://www.accelacomm.com/jaw/sdnl/114/51450054/
___
Nagios-users mailing list
Nagios-users@lists.sourceforge.net
https://lists.sourceforge.net/lists/listinfo/nagios-users
::: Please include Nagios version, plugin version (-v) and OS when reporting 
any issue. 
::: Messages without supporting info will risk being sent to /dev/null


Re: [Nagios-users] timeperiod definition for election day?

2011-12-05 Thread Jim Avery
On 5 December 2011 20:31, Paul M. Dubuc w...@paul.dubuc.org wrote:
 I didn't see this in the documentation, but I wonder if there is a way to
 specify a timeperiod for the first weekday after another weekday.  For
 example, election day in the U.S. is on the 1st Tuesday after the 1st Monday
 of November.  We have a similar need do define a timeperiod for the 1st Sunday
 after the 1st Saturday of every month.

 Must we do this by entering all the specific dates for these in the coming
 year(s), or is there a simpler, no maintenance way of doing it?

 Thanks,
 Paul Dubuc

I was pondering about this sort of thing the other day when I was
updating our bank holiday list.  I think what's really needed is an
equivalent of ntp, but for bank holidays so you have have a central
repository of bank holiday information and have any servers that are
interested poll it once in a while.  That would be nothing to do with
Nagios per se of course, but would be useful to Nagios and lots of
other applications.

Maybe someone has already done such a thing and I've not heard of it yet?

--
All the data continuously generated in your IT infrastructure 
contains a definitive record of customers, application performance, 
security threats, fraudulent activity, and more. Splunk takes this 
data and makes sense of it. IT sense. And common sense.
http://p.sf.net/sfu/splunk-novd2d
___
Nagios-users mailing list
Nagios-users@lists.sourceforge.net
https://lists.sourceforge.net/lists/listinfo/nagios-users
::: Please include Nagios version, plugin version (-v) and OS when reporting 
any issue. 
::: Messages without supporting info will risk being sent to /dev/null


Re: [Nagios-users] Passive monitoring multiple servers with 1 NAT IP

2011-12-03 Thread Jim Avery
On 3 December 2011 17:46,  a.sm...@ukgrid.net wrote:
   ok great if its possible, however I did try setting this up and it
 didnt seem to work.
 I am using the NSClient++ on Windows servers, and under the NSCA
 section I have defined what is described as LOCAL HOST NAME
 correctly on each client. However when both clients are configured on
 the Nagios server updates are received to one and not the other, which
 makes me think its just asscoiating all inbound messages with the
 first matching IP (I have defined each host with an IP, I guess that
 is required too isnt it?),

The 'address' directive in the host definition is mandatory, yes, but
you can put anything in there you like.  If you are not doing any
active checks (and the active checks you might end up doing are
check_dummy anyway), then you can put what you like in there - it
won't be used for anything.

 If it isn't to do with the host definition having an IP defined then
 maybe I made some other config mistake, I can double check it all on
 Monday.

It sounds like it's probably some simple problem with the config on
the host which isn't working, yes.  Sometimes it helps to get nsca (on
the Nagios server) to write some debug to syslog.  I can't remember
off-hand how to do that, but it's probably found in
/usr/local/nagios/etc/nsca.ini or similar or if not, in the xinetd
entry for nsca which if I recall is in /etc/xinetd.d/nsca .

Check the nsclient.log file on the Windows system too, and make sure
there's nothing obvious in there.

Upper/Lower case is important for the host name so what you have in
the Nagios host definition must match what the agent is sending.

--
All the data continuously generated in your IT infrastructure 
contains a definitive record of customers, application performance, 
security threats, fraudulent activity, and more. Splunk takes this 
data and makes sense of it. IT sense. And common sense.
http://p.sf.net/sfu/splunk-novd2d
___
Nagios-users mailing list
Nagios-users@lists.sourceforge.net
https://lists.sourceforge.net/lists/listinfo/nagios-users
::: Please include Nagios version, plugin version (-v) and OS when reporting 
any issue. 
::: Messages without supporting info will risk being sent to /dev/null


Re: [Nagios-users] Passive monitoring multiple servers with 1 NAT IP

2011-12-02 Thread Jim Avery
On 2 December 2011 18:14,  a.sm...@ukgrid.net wrote:
 Hi,

 in the case where I want to monitor several servers in a remote office where
 all outbound traffic from that office originates from a single IP (NAT) is
 there any way I can monitor those with Nagios without putting a Nagios
 server in the remote office?
 On the face of it it would seem impossible as Nagios identifies hosts by
 their IP (and therefore each needs a unique IP) but would be good to get
 that confirmed or otherwise,

Yes you can do that no problem.  Send the checks to the Nagios server
using send_nsca and configure all the hosts in Nagios with passive
host and service checks.  You can use freshness checking to alert you
if no checks have been received at all lately from the remote host.
The send_nsca transmission identifies the host to Nagios so it won't
matter if the address is NATted.

Note that for the active check in Nagios you will need to use
check_dummy or similar so that if the freshness check fails then
Nagios will run this and alert you to the fact the freshness check has
failed.  I normally use something like ..

  check_dummy!3 UNKNOWN: Nagios didn't receive a check result lately
from the server!

You haven't said if the servers are Unix, Windows or what.  If
Windows, then it's easy to configure the NSClient++ agent to send
checks using NSCA.  If Unix or linux you will need to install
send_nsca and the relevant plugins and run the checks from cron, maybe
using the nsca_wrapper script which you'll find on Nagios Exchange.

You will of course need to configure NSCA on your Nagios server to
receive the incoming checks.

hth,

Jim

--
All the data continuously generated in your IT infrastructure 
contains a definitive record of customers, application performance, 
security threats, fraudulent activity, and more. Splunk takes this 
data and makes sense of it. IT sense. And common sense.
http://p.sf.net/sfu/splunk-novd2d
___
Nagios-users mailing list
Nagios-users@lists.sourceforge.net
https://lists.sourceforge.net/lists/listinfo/nagios-users
::: Please include Nagios version, plugin version (-v) and OS when reporting 
any issue. 
::: Messages without supporting info will risk being sent to /dev/null


Re: [Nagios-users] monitor printer from Windows server

2011-12-01 Thread Jim Avery
On 1 December 2011 12:39, James Osbourn james.osbo...@citrix.com wrote:
 I have a Windows 7 server acting as a print server and I would like to check 
 the status of these printers and make sure that they are still online etc.

 I cannot seem to find a way of checking printers shared from a Windows 
 machine, does anyone have any suggestions or references that I could use.

 Thanks

 James


I query the windows performance counter for number of print jobs
spooling.  If a printer breaks then usually this counter starts
ramping up quite rapidly as more and more print jobs get stuck in the
queue.

For example, the [external alias] section of your nsc.ini (assuming
you use NSClient++), you can have:-

alias_CheckCounter-PrintQJobs=CheckCounter Counter:jobs=\Print
Queue(_Total)\Jobs ShowAll MaxWarn=250 MaxCrit=500

Then your service definition would look something like this:-

define service {
host_name   printserver1,printserver2
service_description PrintQJobs
use srv-pnp,generic-service
check_command   check_nrpe!-c PrintQJobs
max_check_attempts  6
check_interval  15
retry_interval  5
contact_groups  notify-admins
notes   Records the number of Active
Jobs on a Print Server
register1
}

And the command definition I use is just a generic one for any nrpe
check like so:-

define command {^M
command_namecheck_nrpe
command_line$USER1$/check_nrpe -H
$HOSTADDRESS$ -u $ARG1$
register1
}

Our servers aren't quite as recent as Windows 7, but I guess the
counter would still be the same.

I hope that helps.

Jim

--
All the data continuously generated in your IT infrastructure 
contains a definitive record of customers, application performance, 
security threats, fraudulent activity, and more. Splunk takes this 
data and makes sense of it. IT sense. And common sense.
http://p.sf.net/sfu/splunk-novd2d
___
Nagios-users mailing list
Nagios-users@lists.sourceforge.net
https://lists.sourceforge.net/lists/listinfo/nagios-users
::: Please include Nagios version, plugin version (-v) and OS when reporting 
any issue. 
::: Messages without supporting info will risk being sent to /dev/null


Re: [Nagios-users] Monitoring a Microsoft SQL Server

2011-11-17 Thread Jim Avery
On 17 November 2011 15:17, Kaplan, Andrew H. ahkap...@partners.org wrote:
 Hi there --

 Thanks for your reply. I tried your suggestion on the two plugins, and here 
 are
 the results.

 When the command syntax for check_mssql was the following:

 ./check_mssql -H ip address -U domain\\username -P password

 or

 ./check_mssql -H ip address -U 'domain\\username' -P password

 The error message displayed on-screen was:

 UNKNOWN: Invalid characters in the username.


The check_mssql plugin is way too fussy about what characters it will
allow in host names and user names.  On my system I edited the plugin
to remove the if/else sections entirely below the comments //
Validate the hostname and // Validate the username.

hth,

Jim

--
All the data continuously generated in your IT infrastructure 
contains a definitive record of customers, application performance, 
security threats, fraudulent activity, and more. Splunk takes this 
data and makes sense of it. IT sense. And common sense.
http://p.sf.net/sfu/splunk-novd2d
___
Nagios-users mailing list
Nagios-users@lists.sourceforge.net
https://lists.sourceforge.net/lists/listinfo/nagios-users
::: Please include Nagios version, plugin version (-v) and OS when reporting 
any issue. 
::: Messages without supporting info will risk being sent to /dev/null


Re: [Nagios-users] nagios timeout checks

2011-11-11 Thread Jim Avery
On 11 November 2011 16:11, Gustavo g.macha...@gmail.com wrote:

 hello,
 this is my first email
 I've just installed nagios 3.2.3
 And i would like to know if is there a way to configurate nagios to send 
 time-out cheks to other group of users.
 The problem is that if some times the monitoring service works fine... but 
 once or twice a day it does time out exception.
 Can i config a way to if the monitoring works fine send normally the email, 
 if it happends a time out exception send this erro for me to see what 
 happends.


 thakou in advanced and sorry for my bad english,  hope you undarstand   :)


If the timeout errors result in an UNKNOWN state, then you can
configure your ordinary users (for example) to receive only WARNING,
CRITICAL and RECOVERY notifications, but your Nagios administrator
users only to receive UNKNOWN and FLAPPING notifications.  That is set
for all notifications for that contact though.

For example:

define contact {
  contact_name  luciano_lanza
  aliasordinary user Luciano Lanza
  host_notification_options  d,r
  service_notification_options  w,c,r
  email  lla...@hotmael.com
  }

define contact {
  contact_name  gustavo_machado
  alias Nagios Admin G. Machado
  host_notification_options  u,f
  service_notification_options  u,f
  email  gmach...@hotmael.com
  }


Note that the check_nt plugin will only result in UNKNOWN on timeout
if you specify the -u or --unknown-timeout option.

Unfortunately some other plugins do set status to UNKNOWN on timeout
but others don't.

With some checks, it helps to set max_check_attempts to 3 or more so
that if the plugin times out only once or twice then Nagios will not
send a notification.


Cheers, and welcome!

Jim

--
RSA(R) Conference 2012
Save $700 by Nov 18
Register now
http://p.sf.net/sfu/rsa-sfdev2dev1
___
Nagios-users mailing list
Nagios-users@lists.sourceforge.net
https://lists.sourceforge.net/lists/listinfo/nagios-users
::: Please include Nagios version, plugin version (-v) and OS when reporting 
any issue. 
::: Messages without supporting info will risk being sent to /dev/null


Re: [Nagios-users] Checking External URLs

2011-11-08 Thread Jim Avery
On 8 November 2011 22:18, Corey Quinn co...@sequestered.net wrote:
 What's the canonically proper way to check an external website (that I 
 don't control) within Nagios?  WHat I've done historically has been to define 
 a dummy urlhost that I then hang various check_http checks off of, but that 
 seems wrong somehow.

I don't think there is a 'proper' way.  I would think you're probably
best off thinking about the parent relationships though.  If you lose
contact with all your external websites, then what is the furthest
thing that might have gone down?  Is that the customer-facing
interface of your ISP's router maybe?  I would hang your external web
service checks on that.

Obviously if you don't own these external sites then it's polite to
check them infrequently and ask the site owner if it's okay.

Personally I think there's no real substitute for setting up a
monitoring system or two elsewhere out there in the internet if you
want to be really sure a site is available to customers.

--
RSA(R) Conference 2012
Save $700 by Nov 18
Register now
http://p.sf.net/sfu/rsa-sfdev2dev1
___
Nagios-users mailing list
Nagios-users@lists.sourceforge.net
https://lists.sourceforge.net/lists/listinfo/nagios-users
::: Please include Nagios version, plugin version (-v) and OS when reporting 
any issue. 
::: Messages without supporting info will risk being sent to /dev/null


Re: [Nagios-users] check_nt wrapper

2011-11-07 Thread Jim Avery
On 4 November 2011 14:18, mazar ma...@autistici.org wrote:
 Hi list,

 I've put together a wrapper for the Disk Reads/sec and Disk Writes/sec
 counters, my goal is to have an output as follow:

 ./check_diskio_win.sh 1.1.1.1 C

 Disk Reads/sec is 5 ; Disk Writes/sec is 25

 I'm able to accomplish this via script, but when I give it to nagios, as
 check_command the output (web interface) is as follow:

 Disk Reads/sec is : Disk Writes/sec is

 what I do in the script is basically the below command and some parsing
 (that works from shell script):

 check_nt -H $1 -v COUNTER  -l \\PhysicalDisk(0 $2:)\\Disk
 Reads/sec,PhysicalDisk(0 $2:) Disk Reads/sec is %.f 

My guess is the shell is interpreting the % character in some way.
Try escaping it like so ..
  Reads/sec is \%.f 

but note that sometimes you need to escape a few times, as it might be
some other level in the process that needs the backslash to appear
intact.  Try with two or even three backslashes like so..
  Reads/sec is \\\%.f 

The most number of backslashes I have needed in a similar situation was five!

hth

--
RSA(R) Conference 2012
Save $700 by Nov 18
Register now
http://p.sf.net/sfu/rsa-sfdev2dev1
___
Nagios-users mailing list
Nagios-users@lists.sourceforge.net
https://lists.sourceforge.net/lists/listinfo/nagios-users
::: Please include Nagios version, plugin version (-v) and OS when reporting 
any issue. 
::: Messages without supporting info will risk being sent to /dev/null


Re: [Nagios-users] Nagios Web Interface does not notify when nagios service is killed!

2011-10-28 Thread Jim Avery
On 28 October 2011 18:55, Ashwin discoverash...@yahoo.co.uk wrote:
 Hi,

 I am working as UNIX administrator in Data Centre having more than few
 hundred servers(RHEL 5.5/6). I have installed Nagios (core) 3.3.1 and Nagios
 Plugins 1.4.15 and NRPE (Nagios Remote Plugin Executor) 2.12 to monitor the
 all servers in data centre mentioned above.

 When nagios process is killed manually at the nagios monitoring server, the
 nagios web interface does not automatically notify about the nagios process
 been killed (unless the service is explicitly stopped).

 This gives impression to support team looking at the web interface that
 everything is working fine when nagios process itself is killed/dead. I am
 new to nagios/RHEL, can you please give me some pointers to solve this
 issue.


In the (more years than I care to remember) that I've been running a
Nagios system, I've not had a problem with the nagios daemon being
killed manually.  So long as you stop/start it using the init script
the front end will make it pretty obvious the daemon is stopped.  If
your admins are fool enough to kill stuff off willy-nilly you need to
lock them out of your Nagios system, and any other system that's
important to you for that matter.

If you really want to know the daemon is killed, then simply knock up
a script, ideally running on another server, which queries your Nagios
server using snmp or ssh or whatever to see if the nagios daemon is
running and emails or SMSs you if it isn't.  The check_snmp_process
plugin could be useful for that task.
http://nagios.manubulon.com/snmp_process.html

hth,

Jim

--
The demand for IT networking professionals continues to grow, and the
demand for specialized networking skills is growing even more rapidly.
Take a complimentary Learning@Cisco Self-Assessment and learn 
about Cisco certifications, training, and career opportunities. 
http://p.sf.net/sfu/cisco-dev2dev
___
Nagios-users mailing list
Nagios-users@lists.sourceforge.net
https://lists.sourceforge.net/lists/listinfo/nagios-users
::: Please include Nagios version, plugin version (-v) and OS when reporting 
any issue. 
::: Messages without supporting info will risk being sent to /dev/null


Re: [Nagios-users] Average Check latency and execution time growth - 3.2.3

2011-10-07 Thread Jim Avery
On 3 October 2011 04:36, Stuart Browne stuart.bro...@ausregistry.com.au wrote:
 Hi,

 I know this topic has been covered many times, but I've tried those tweaks 
 and I have the remaining issue.

 After a few days, the latency on checks explodes.  It goes along quite 
 happily with small values, then after (about) 3 days, the values rise quite 
 sharply.  I've recently been graphing performance statistics (nagiostats, 
 mrtg) and as you can see by the two attachments (day, week), it's rather 
 surprising.


I'm sorry I can't shed much light on it, but I've seen the same
behaviour myself, albeit on my system the service check latency
wouldn't start increasing until after a week or two but you're right -
the rate of increase when it starts is quite alarming.  I've recently
culled a lot of checks from the system which has ameliorated the issue
for the time being, but it would be good to get it fixed properly.

--
All of the data generated in your IT infrastructure is seriously valuable.
Why? It contains a definitive record of application performance, security
threats, fraudulent activity, and more. Splunk takes this data and makes
sense of it. IT sense. And common sense.
http://p.sf.net/sfu/splunk-d2dcopy2
___
Nagios-users mailing list
Nagios-users@lists.sourceforge.net
https://lists.sourceforge.net/lists/listinfo/nagios-users
::: Please include Nagios version, plugin version (-v) and OS when reporting 
any issue. 
::: Messages without supporting info will risk being sent to /dev/null


Re: [Nagios-users] Historical Reports

2011-09-20 Thread Jim Avery
On 19 September 2011 11:43, Lingan Vairavamoorthy
lin...@avantiagroup.co.uk wrote:
 I’ve looked through the posts and haven’t seen anything on full logging
 information.



 What we are looking for is the ability to view all the checks that have
 occurred on a host/service regardless if there was a change of state.



 This will enable us to do graphing on performance data and look at trends
 when states would not have changed.



 i.e. We are checking a response to a web page and we only warn on large
 delays – but we would like to actually record all the timings it takes to
 visit that web page regardless if the state changed from OK to WARN. So that
 we would have a entry in the historical logs for every check. Is that
 feasible?


Nagios itself doesn't usually write the check result to the log if the
service is in a hard state and the state hasn't changed.  You can
configure it to do that, but for best performance I wouldn't recommend
it.

To store, process and graph performance data, you are probably best
off using PNP4Nagios http://docs.pnp4nagios.org/pnp-0.6/start .  It's
quite easy to extract data from the .rrd databases created by
PNP4Nagios using rrdexport if you need it in xml format or similar.

--
All the data continuously generated in your IT infrastructure contains a
definitive record of customers, application performance, security
threats, fraudulent activity and more. Splunk takes this data and makes
sense of it. Business sense. IT sense. Common sense.
http://p.sf.net/sfu/splunk-d2dcopy1
___
Nagios-users mailing list
Nagios-users@lists.sourceforge.net
https://lists.sourceforge.net/lists/listinfo/nagios-users
::: Please include Nagios version, plugin version (-v) and OS when reporting 
any issue. 
::: Messages without supporting info will risk being sent to /dev/null


Re: [Nagios-users] Historical Reports

2011-09-20 Thread Jim Avery
On 20 September 2011 01:16, Samuel Kidman samuel.kid...@panres.com wrote:
 The only gripe I have with it is the graphs are rarely ever up-to-date
 when you get them- you're always about 5-10 minutes behind. It would be
 nice if there was a way of triggering the  databases to be updated and
 graphs to be drawn in their most up-to-date form as soon as they are
 requested.

You might need to look at some of the many ways you can speed up
PNP4Nagios processing.  I agree the graphs aren't updated instantly
but 10-20 seconds should be a more typical time to wait for the graph
to be updated, not 5-10 minutes even in a fairly large Nagios system.

At the most basic level, how often Nagios will process performance
data determined by these two directives in nagios.cfg:

# HOST AND SERVICE PERFORMANCE DATA FILE PROCESSING INTERVAL
# These options determine how often (in seconds) the host and service
# performance data files are processed using the commands defined
# below.  A value of 0 indicates the files should not be periodically
# processed.

host_perfdata_file_processing_interval=15
service_perfdata_file_processing_interval=10

--
All the data continuously generated in your IT infrastructure contains a
definitive record of customers, application performance, security
threats, fraudulent activity and more. Splunk takes this data and makes
sense of it. Business sense. IT sense. Common sense.
http://p.sf.net/sfu/splunk-d2dcopy1
___
Nagios-users mailing list
Nagios-users@lists.sourceforge.net
https://lists.sourceforge.net/lists/listinfo/nagios-users
::: Please include Nagios version, plugin version (-v) and OS when reporting 
any issue. 
::: Messages without supporting info will risk being sent to /dev/null


Re: [Nagios-users] suddenly, cgi stopped working...

2011-09-16 Thread Jim Avery
On 16 September 2011 11:25, Metron 6 (six) metr...@gmail.com wrote:
 Hello Jim,

 this directive is not in nagios.cfg. actually there is no such
 directive in any conf file...
 when i issue into /etc/nagios3 this command grep status_file *  i get no 
 results

 i did this
 find / -name status.dat
 /var/lib/nagios3/status.dat

 and found that it has the right permissions
 -rw-rw-r--  1 nagios www-data    223007 2011-09-16 13:20 status.dat

 so, whats next ?

I don't know Centreon, but in a nagios core install I would expect the
.cgi should read the nagios.cfg file to determine the location of
status.dat.  I would add the directive
status_file=/var/lib/nagios3/status.dat anyway and see if that helps
the .cgi to find the status file.

(in a default Nagios core install) Your cgi.cfg file should point to
the location of nagios.cfg like so:
  # MAIN CONFIGURATION FILE
  # This tells the CGIs where to find your main configuration file.
  # The CGIs will read the main and host config files for any other
  # data they might need.
  main_config_file=/usr/local/nagios/etc/nagios.cfg


I would raise this on the relevant Centreon forum as I would guess
you're more likely to find people there who understand the
peculiarities of Centreon's implementation of nagios.  Some of the
newer front-ends don't use status.dat to get status information from
Nagios - I wouldn't know if Centreon is one of those.

--
BlackBerryreg; DevCon Americas, Oct. 18-20, San Francisco, CA
http://p.sf.net/sfu/rim-devcon-copy2
___
Nagios-users mailing list
Nagios-users@lists.sourceforge.net
https://lists.sourceforge.net/lists/listinfo/nagios-users
::: Please include Nagios version, plugin version (-v) and OS when reporting 
any issue. 
::: Messages without supporting info will risk being sent to /dev/null


Re: [Nagios-users] running an external script for sms notification

2011-09-15 Thread Jim Avery
On 15 September 2011 10:35, Metron 6 (six) metr...@gmail.com wrote:
 hello all,

 i have a shell script which takes two arguments, the mobile phone
 number and the text message, which sends sms notifications.
 for example /usr/local/bin/sendsms 69 host problem

 when i debug Nagios, i can see this into the log file

 [1316012199.774170] [256.1] [pid=17625] Running command
 '/usr/lib/nagios/plugins/sendsms 6944420xxx Type:PROBLEM, Host:
 00-dummy_2, State: DOWN, Address: 194.145.127.44, Info: Unknown ping
 number, Date/Time: 14-09-2011'...

 when i run from a terminal the command
 /usr/lib/nagios/plugins/sendsms 6944420xxx Type:PROBLEM, Host:
 00-dummy_2, State: DOWN, Address: 194.145.127.44, Info: Unknown ping
 number, Date/Time: 14-09-2011
 sms message arrives normally

 this is how i include the command into nagios conf files..

 define command{
       command_name                    host-notify-by-sms
       command_line                    /usr/lib/nagios/plugins/sendsms
 $CONTACTPAGER$ Type:$NOTIFICATIONTYPE$, Host: $HOSTNAME$, State:
 $HOSTSTATE$, Address: $HOSTADDRESS$, Info: $HOSTOUTPUT$, Date/Time:
 $DATE$
 }

 any help/suggestions ?

Looks ok to me.  Am I right in assuming that when Nagios runs the
command you receive no SMS at all?  Or do you receive something but in
the wrong format?

When you run the command from the command-line, are you logged in as
'nagios'?  If not it could be a simple permissions problem.

--
Doing More with Less: The Next Generation Virtual Desktop 
What are the key obstacles that have prevented many mid-market businesses
from deploying virtual desktops?   How do next-generation virtual desktops
provide companies an easier-to-deploy, easier-to-manage and more affordable
virtual desktop model.http://www.accelacomm.com/jaw/sfnl/114/51426474/
___
Nagios-users mailing list
Nagios-users@lists.sourceforge.net
https://lists.sourceforge.net/lists/listinfo/nagios-users
::: Please include Nagios version, plugin version (-v) and OS when reporting 
any issue. 
::: Messages without supporting info will risk being sent to /dev/null


Re: [Nagios-users] suddenly, cgi stopped working...

2011-09-15 Thread Jim Avery
On 15 September 2011 13:42, Metron 6 (six) metr...@gmail.com wrote:
 hello all,

 I have nagios installed with centreon interface to add hosts, services, etc..
 Suddenly Nagios stopped showing data, in web interface.

 i get the message:
 Whoops!
 Error: Could not read host and service status information!

 it was working this morning. Nagios is running, as far as i can see
 from the log file.

 please, help..

The host and service information which the cgi is trying to read is in
the status.dat file.  Where that is depends on how you installed
Nagios, but is specified in nagios.cfg, the status_file= directive.
In a default install from tarballs it will be
/usr/local/nagios/var/status.dat .  Check if it exists and what the
ownership/permissions are.  It should be readable by your apache
daemon.

I would stop nagios:-
/etc/init.d/nagios stop

Then verify the config:-

/usr/local/nagios/bin/nagios -v /usr/local/nagios/etc/nagios.cfg

Then check there are no nagios processes running (using ps), then
start Nagios again:-
/etc/init.d/nagios start


It's always best to verify the config before restarting or reloading
the Nagios daemon.

I hope that gives you some pointers.

Cheers,

Jim  (Capacity Planner for my sins)

--
Doing More with Less: The Next Generation Virtual Desktop 
What are the key obstacles that have prevented many mid-market businesses
from deploying virtual desktops?   How do next-generation virtual desktops
provide companies an easier-to-deploy, easier-to-manage and more affordable
virtual desktop model.http://www.accelacomm.com/jaw/sfnl/114/51426474/
___
Nagios-users mailing list
Nagios-users@lists.sourceforge.net
https://lists.sourceforge.net/lists/listinfo/nagios-users
::: Please include Nagios version, plugin version (-v) and OS when reporting 
any issue. 
::: Messages without supporting info will risk being sent to /dev/null


Re: [Nagios-users] Nagios.log not updating

2011-09-09 Thread Jim Avery
On 9 September 2011 16:01, Mailing Lists mailing.lists.a...@gmail.com wrote:
 I currently have a centreon / nagios setup with one central centreon box
 with three pollers. The newest poller is working fine with in terms of
 updating the central centreon system. However, the one issue I am running
 into and haven't been able to find an answer to is that the newest poller
 isn't logging to the nagios.log when a service is checked. It logs the
 initial service state and that's it. Any idea what could cause this? I have
 googled for hours.

Have you checked the various option in nagios.cfg?  See:-
http://nagios.sourceforge.net/docs/nagioscore/3/en/configmain.html
log_service_retries and log_passive_checks etc,.

--
Why Cloud-Based Security and Archiving Make Sense
Osterman Research conducted this study that outlines how and why cloud
computing security and archiving is rapidly being adopted across the IT 
space for its ease of implementation, lower cost, and increased 
reliability. Learn more. http://www.accelacomm.com/jaw/sfnl/114/51425301/
___
Nagios-users mailing list
Nagios-users@lists.sourceforge.net
https://lists.sourceforge.net/lists/listinfo/nagios-users
::: Please include Nagios version, plugin version (-v) and OS when reporting 
any issue. 
::: Messages without supporting info will risk being sent to /dev/null


Re: [Nagios-users] Antwort: Re: Antwort: Re: check_snmp_int - No usable data on file

2011-09-05 Thread Jim Avery
On 5 September 2011 07:49,  radu.p...@technomatic.de wrote:
 Hello again Jim,

 I have followed your suggestion and seems better. However, from time to
 time, for short periods ( which take 1-2 checks) the status flaps to unknown
 and afterwards gets back to normal. Any more clues, please?

 Thanks!

The documentation page seems to suggest that the -d option should be a
slightly longer than the check interval.

http://nagios.manubulon.com/snmp_int.html

If there's anything in your nagios system which will cause the nagios
daemon to do something other than scheduling checks for a few seconds,
then with a 10s standard interval it wouldn't take much to throw the
plugin off kilter.  I don't know for sure which activities would do
this, but suspect one would be writing to the status.dat file and
another might be processing the command file.  Having said that, with
the check_snmp_int.pl plugin allowing up to 300% of the -d value
before complaining, it should be fairly forgiving.

--
Special Offer -- Download ArcSight Logger for FREE!
Finally, a world-class log management solution at an even better 
price-free! And you'll get a free Love Thy Logs t-shirt when you
download Logger. Secure your free ArcSight Logger TODAY!
http://p.sf.net/sfu/arcsisghtdev2dev
___
Nagios-users mailing list
Nagios-users@lists.sourceforge.net
https://lists.sourceforge.net/lists/listinfo/nagios-users
::: Please include Nagios version, plugin version (-v) and OS when reporting 
any issue. 
::: Messages without supporting info will risk being sent to /dev/null


Re: [Nagios-users] Antwort: Re: Antwort: Re: Antwort: Re: check_snmp_int - No usable data on file

2011-09-05 Thread Jim Avery
On 5 September 2011 08:13,  radu.p...@technomatic.de wrote:
 So your suggestion is to have the -d option set to 60s for a 20s check
 interval?

No.  The plugin will allow an interval of up to 300% above the -d
value, but only 10% below the -d value.  This implies you could go up
to -d = 21 .  This would mean it will allow a gap of up to 63 seconds
rather than 60.  I suppose to be honest that's not likely to make much
difference to you!

I would look at what it is in your config which might cause the nagios
daemon to stop running scheduled checks for 60 seconds or more.  One
indicator which is useful in assessing this sort of thing is the
service check latency.  Moving status.dat on to a ramdisk might help -
it didn't work for me, as I suffered locking issues, but with a
different Linux distro it might work better.  I was using Ubuntu 8.04
LTS.

Alternatively you could maybe look at running these checks using
check_snmp instead.  The very latest version has the options you need
to record the delta rather than absolute values.

If you can increase the check interval even only a little that would
help.  I use a 5 minute check interval for these interface checks, and
usually only see the same errors whenever I reload the Nagios config.

You might also consider configuring Nagios in a hierarchical setup,
with each slave Nagios server handling fewer checks.  I would think
DNX would be an easy way to set that up ( http://dnx.sourceforge.net/
), but I've not tried it yet myself.

--
Special Offer -- Download ArcSight Logger for FREE!
Finally, a world-class log management solution at an even better 
price-free! And you'll get a free Love Thy Logs t-shirt when you
download Logger. Secure your free ArcSight Logger TODAY!
http://p.sf.net/sfu/arcsisghtdev2dev
___
Nagios-users mailing list
Nagios-users@lists.sourceforge.net
https://lists.sourceforge.net/lists/listinfo/nagios-users
::: Please include Nagios version, plugin version (-v) and OS when reporting 
any issue. 
::: Messages without supporting info will risk being sent to /dev/null


Re: [Nagios-users] Observing latency

2011-09-05 Thread Jim Avery
On 5 September 2011 12:53, Daniel Wittenberg
daniel.wittenberg.r...@statefarm.com wrote:
 We also had to do some tuning with our pnp4nagios for latency.  I ended up 
 using a ramdisk for the perfdata folder (that it uses while processing data, 
 not the rrd's).


Thanks for that - good idea I must try it.  I don't have a problem
with latency at the moment, but I suspect that anything I can do to
improve disk performance will improve the responsiveness of the Nagios
CGIs.

--
Special Offer -- Download ArcSight Logger for FREE!
Finally, a world-class log management solution at an even better 
price-free! And you'll get a free Love Thy Logs t-shirt when you
download Logger. Secure your free ArcSight Logger TODAY!
http://p.sf.net/sfu/arcsisghtdev2dev
___
Nagios-users mailing list
Nagios-users@lists.sourceforge.net
https://lists.sourceforge.net/lists/listinfo/nagios-users
::: Please include Nagios version, plugin version (-v) and OS when reporting 
any issue. 
::: Messages without supporting info will risk being sent to /dev/null


Re: [Nagios-users] check_snmp_int - No usable data on file

2011-09-04 Thread Jim Avery
On 4 September 2011 10:53,  radu.p...@technomatic.de wrote:
 Hello,

 I am using check_snmp_plugin to monitor a couple of switches. The command
 template I use is:

  $USER1$/check_snmp_int.pl -H $HOSTADDRESS$ -C $USER5$ -2 -r -n $ARG1$ -fY
 -kqB -g -w $ARG2$ -c $ARG3$ -d $ARG4$

 And the particular one for a certain port/device is:

 # Monitor bandwidth and port status - port 23
 define service{
         use                        generic-service                ; Inherit
 values from a template
         Host_name                switch1
         Service_description        Port 23.
         Check_command
  check_snmp_int!GigabitEthernet0/23!60,60,100,100,10,10!80,80,200,200,20,20!11
         Normal_check_interval        2                ; Check the service
 every 2 units under normal conditions
         retry_check_interval--1                ; Re-check the service
 every one unit until its final/hard state is determined
         }


 The interval length in nagios.cfg is set to:

 interval_length=10

 Can you please let me know a good value for the -d parameter in order to
 tweak the check_snmp_int.pl script properly?


I believe that with check_interval of 2 minutes, you need -d 120.

With the plugin working in this way, you need to set your
retry_interval to be the same as your check_interval, otherwise the -d
value won't be correct for the interval when the check starts
retrying!

Note that in Nagios 3, the syntax normal_check_interval and
retry_check_interval is deprecated.  The equivalent directives are
now simply check_interval and retry_interval.  See:
http://nagios.sourceforge.net/docs/nagioscore/3/en/objectdefinitions.html#service

If you are still having problems with the No usable data on file
error, make sure the files created by the plugin are writeable by the
user 'nagios'  (if I recall they are under /tmp or maybe /var/tmp - I
don't have my nagios system to hand to check for sure).  If you have
been testing the plugin from the command line while logged in as
another user, it's quite likely the files will have the wrong
ownership/permissions in which case you can simply delete them to let
nagios recreate them next time it runs the plugin.

--
Special Offer -- Download ArcSight Logger for FREE!
Finally, a world-class log management solution at an even better 
price-free! And you'll get a free Love Thy Logs t-shirt when you
download Logger. Secure your free ArcSight Logger TODAY!
http://p.sf.net/sfu/arcsisghtdev2dev
___
Nagios-users mailing list
Nagios-users@lists.sourceforge.net
https://lists.sourceforge.net/lists/listinfo/nagios-users
::: Please include Nagios version, plugin version (-v) and OS when reporting 
any issue. 
::: Messages without supporting info will risk being sent to /dev/null


Re: [Nagios-users] Antwort: Re: check_snmp_int - No usable data on file

2011-09-04 Thread Jim Avery
On 4 September 2011 11:42,  radu.p...@technomatic.de wrote:
 Hello Jim and thanks for your answer!

 I have double checked the files in /tmp created by the script and they are
 all owned by nagios group and nagios user, so fully writeable.

 The actual check interval of the port is 20 seconds ( I check every 2 units
 and the time unit is 10 seconds in the nagios.cfg). The correct value for
 the plugin time is in this case 20s?

I see, yes, in that case you should use -d 20.

 I will give it a try with making the normal check interval and retry
 interval at the same value.

I think that will most likely solve your problem.

Cheers,

Jim

--
Special Offer -- Download ArcSight Logger for FREE!
Finally, a world-class log management solution at an even better 
price-free! And you'll get a free Love Thy Logs t-shirt when you
download Logger. Secure your free ArcSight Logger TODAY!
http://p.sf.net/sfu/arcsisghtdev2dev
___
Nagios-users mailing list
Nagios-users@lists.sourceforge.net
https://lists.sourceforge.net/lists/listinfo/nagios-users
::: Please include Nagios version, plugin version (-v) and OS when reporting 
any issue. 
::: Messages without supporting info will risk being sent to /dev/null


Re: [Nagios-users] Notifications for Services that are Up

2011-09-02 Thread Jim Avery
On 2 September 2011 15:59, Michael Loiselle mloise...@chan-nh.org wrote:
 Is it possible to receive a notification for a specific
 service that is in the “up” position?

If you really must do that, then assuming the check is an active one,
then it should be pretty trivial to run the plugin from cron and email
you whatever based on the exit code.  All that is done completely
outside Nagios.

--
Special Offer -- Download ArcSight Logger for FREE!
Finally, a world-class log management solution at an even better 
price-free! And you'll get a free Love Thy Logs t-shirt when you
download Logger. Secure your free ArcSight Logger TODAY!
http://p.sf.net/sfu/arcsisghtdev2dev
___
Nagios-users mailing list
Nagios-users@lists.sourceforge.net
https://lists.sourceforge.net/lists/listinfo/nagios-users
::: Please include Nagios version, plugin version (-v) and OS when reporting 
any issue. 
::: Messages without supporting info will risk being sent to /dev/null


Re: [Nagios-users] Observing latency

2011-08-31 Thread Jim Avery
On 31 August 2011 09:13, trm asn trm.nag...@gmail.com wrote:

 Please suggest , how I can reduce the latency ..


One not-so-obvious thing I found that reduces latency on my
heavily-loaded system is to stop and then start the Nagios daemon
(don't just do a reload) once a week or so.

--
Special Offer -- Download ArcSight Logger for FREE!
Finally, a world-class log management solution at an even better 
price-free! And you'll get a free Love Thy Logs t-shirt when you
download Logger. Secure your free ArcSight Logger TODAY!
http://p.sf.net/sfu/arcsisghtdev2dev
___
Nagios-users mailing list
Nagios-users@lists.sourceforge.net
https://lists.sourceforge.net/lists/listinfo/nagios-users
::: Please include Nagios version, plugin version (-v) and OS when reporting 
any issue. 
::: Messages without supporting info will risk being sent to /dev/null


Re: [Nagios-users] how to change nagiosadmin password - forgot it

2011-08-26 Thread Jim Avery
On 26 August 2011 17:06, Stan Laughlin stan.laugh...@gmail.com wrote:
 Folks,
 I have a Nagios installation on a RedHat enterprise system.
 I deployed and successfully logged to the Nagios system several months ago.
 Then was pulled to other projects.  Now I can get back to the Nagios thing.

 I happily displays the web site login.
 I can not remember the nagiosadmin password.

 I can find sites telling me how to create a new account w/a related password
 but I don't think I need a new account.  I just need the password.

 Is there a method for simply changing the password for the existing
 nagiosadmin account?



htpasswd /usr/local/nagios/etc/htpasswd.users nagiosadmin

--
EMC VNX: the world's simplest storage, starting under $10K
The only unified storage solution that offers unified management 
Up to 160% more powerful than alternatives and 25% more efficient. 
Guaranteed. http://p.sf.net/sfu/emc-vnx-dev2dev
___
Nagios-users mailing list
Nagios-users@lists.sourceforge.net
https://lists.sourceforge.net/lists/listinfo/nagios-users
::: Please include Nagios version, plugin version (-v) and OS when reporting 
any issue. 
::: Messages without supporting info will risk being sent to /dev/null


Re: [Nagios-users] Nagios father and nagios son concept

2011-08-19 Thread Jim Avery
On 19 August 2011 01:19, Saulo Zimbaro saulo.zimb...@acotel.com wrote:
 I have more then one data centers to monitoring. I´d like to know if there
 is a way to use various nagios instances and one main nagios to centralize
 information.

The relevant page in the documentation is:

http://nagios.sourceforge.net/docs/nagioscore/3/en/distributed.html

Which should give you a start.  There are a few other solutions out
there too, in particular op5's Merlin gets a lot of mention here.

http://www.op5.org/community/plugin-inventory/op5-projects/merlin

Personally I'm still squeezing all the performance I can from a single
server (with PNP and MySQL on a second one), but it won't be long
until I need to do something similar.

--
Get a FREE DOWNLOAD! and learn more about uberSVN rich system, 
user administration capabilities and model configuration. Take 
the hassle out of deploying and managing Subversion and the 
tools developers use with it. http://p.sf.net/sfu/wandisco-d2d-2
___
Nagios-users mailing list
Nagios-users@lists.sourceforge.net
https://lists.sourceforge.net/lists/listinfo/nagios-users
::: Please include Nagios version, plugin version (-v) and OS when reporting 
any issue. 
::: Messages without supporting info will risk being sent to /dev/null


Re: [Nagios-users] Deleting old comments

2011-08-19 Thread Jim Avery
Ok, I've written the script and submitted it to Nagios Exchange - it
should appear there in the next few days.  I doubt it's the prettiest
code you've seen as this is probably the first perl script I've
written from scratch but it works pretty well (for me anyway - ymmv).

I haven't yet tried to make it do the 'more than X number per host'
thing.  That's just too clever by half! :-)

--
Get a FREE DOWNLOAD! and learn more about uberSVN rich system, 
user administration capabilities and model configuration. Take 
the hassle out of deploying and managing Subversion and the 
tools developers use with it. http://p.sf.net/sfu/wandisco-d2d-2
___
Nagios-users mailing list
Nagios-users@lists.sourceforge.net
https://lists.sourceforge.net/lists/listinfo/nagios-users
::: Please include Nagios version, plugin version (-v) and OS when reporting 
any issue. 
::: Messages without supporting info will risk being sent to /dev/null


Re: [Nagios-users] using check_snmp to chart page counts

2011-08-15 Thread Jim Avery
On 15 August 2011 01:24, Phil Kennedy
phillip.kenn...@yankeeairmuseum.org wrote:
 I'm using check_snmp to monitor page counts on various printers. The
 output is returning the correct values, and PNP is charting, but it's
 showing the differences in page counts per polling interval, rather than
 an overall chart of growth (e.g., the chart says there are 100 pages,
 then an hour from now it bumps up to 110, and so on.) From what I've
 found, this is due to the output from SNP being a counter, rather than
 a gauge, is this something I can circumvent in the name of pretty
 charts and graphs?

 The command I'm using in the commands.cfg is as follows:

 $USER1$/check_snmp -H $HOSTADDRESS$ -C $ARG1$ -o
 .1.3.6.1.2.1.43.10.2.1.4.1.1 -l 'page count' -u 'pages'

The very latest version of check_snmp has a --rate option which is
just what you need for this sort of thing.

http://nagiosplugins.org/man/check_snmp

--
uberSVN's rich system and user administration capabilities and model 
configuration take the hassle out of deploying and managing Subversion and 
the tools developers use with it. Learn more about uberSVN and get a free 
download at:  http://p.sf.net/sfu/wandisco-dev2dev
___
Nagios-users mailing list
Nagios-users@lists.sourceforge.net
https://lists.sourceforge.net/lists/listinfo/nagios-users
::: Please include Nagios version, plugin version (-v) and OS when reporting 
any issue. 
::: Messages without supporting info will risk being sent to /dev/null


Re: [Nagios-users] checking service once a day

2011-08-12 Thread Jim Avery
On 12 August 2011 08:36, Silver Salonen sil...@serverock.ee wrote:
 Hello.

 I want to check a service only once a day. So I tried this configuration:

 define timeperiod {
     timeperiod_name once-a-day
     alias       On mornings
     monday      08:00-08:30
     tuesday     08:00-08:30
     wednesday   08:00-08:30
     thursday    08:00-08:30
     friday      08:00-08:30
     saturday    08:00-08:30
     sunday      08:00-08:30
 }

 define service {
     use generic-service
     check_period        once-a-day
     normal_check_interval  1440        ; 24 hours
     ...
 }

 Now when I check the service's next schedule time, Nagios is still
 showing that the next check is scheduled to 24h after the previous check
 and at 22:53 (pm). Restarting Nagios does not change that. Any tips for
 what's wrong in the configuration?


If Nagios wants to schedule a check at a certain time, but it's not in
the timeperiod then it won't run it.

If you don't mind what time of day your check is run, then use the
standard 24x7 timeperiod.

If you want a check to run at a specific time of day, you need to run
it from cron so it submits the check result to Nagios as a passive
check.

http://nagios.sourceforge.net/docs/nagioscore/3/en/passivechecks.html

Alternatively, if you don't mind your check being run a couple of
times each day, I would think you could leave your config as it is,
but set the normal_check_interval to 12 (minutes) or so to give it a
good chance of being scheduled within your 30-minute window.
Personally I would use cron, but it can be a bit fiddly to set up the
first time you try it.

hth,

Jim

--
Get a FREE DOWNLOAD! and learn more about uberSVN rich system, 
user administration capabilities and model configuration. Take 
the hassle out of deploying and managing Subversion and the 
tools developers use with it. 
http://p.sf.net/sfu/wandisco-dev2dev
___
Nagios-users mailing list
Nagios-users@lists.sourceforge.net
https://lists.sourceforge.net/lists/listinfo/nagios-users
::: Please include Nagios version, plugin version (-v) and OS when reporting 
any issue. 
::: Messages without supporting info will risk being sent to /dev/null


Re: [Nagios-users] checking service once a day

2011-08-12 Thread Jim Avery
On 12 August 2011 10:37, Silver Salonen sil...@serverock.ee wrote:

 So I guess documentation is wrong then?

That's what I would think, yes.  I don't think what Nagios does is
anything like as clever as the documentation would imply, although for
checks with small check_interval values it might seem like it is.
Having said that, I've not dared to attempt reading the source code
myself!

--
Get a FREE DOWNLOAD! and learn more about uberSVN rich system, 
user administration capabilities and model configuration. Take 
the hassle out of deploying and managing Subversion and the 
tools developers use with it. 
http://p.sf.net/sfu/wandisco-dev2dev
___
Nagios-users mailing list
Nagios-users@lists.sourceforge.net
https://lists.sourceforge.net/lists/listinfo/nagios-users
::: Please include Nagios version, plugin version (-v) and OS when reporting 
any issue. 
::: Messages without supporting info will risk being sent to /dev/null


Re: [Nagios-users] checking service once a day

2011-08-12 Thread Jim Avery
On 12 August 2011 15:03, Silver Salonen sil...@serverock.ee wrote:

 Ah yes, previously I just restarted Nagios every day (from crontab) so that
 the checks would be re-scheduled, but in the new installation Nagios does
 not re-schedule the checks on startup. Is there an option for that? I don't
 seem to find it...


It's been a while since I've played with any of those options myself.
I think you'll find it's this directive in nagios.cfg which controls
whether Nagios re-schedules checks on startup ..

# USE RETAINED SCHEDULING INFO
# This setting determines whether or not Nagios will retain
# the scheduling info (next check time) for hosts and services
# based on the values saved in the retention file.  If you
# If you want to use retained scheduling info, set this
# value to 1.  If not, set this value to 0.

use_retained_scheduling_info=1


Cheers,

Jim

--
Get a FREE DOWNLOAD! and learn more about uberSVN rich system, 
user administration capabilities and model configuration. Take 
the hassle out of deploying and managing Subversion and the 
tools developers use with it. 
http://p.sf.net/sfu/wandisco-dev2dev
___
Nagios-users mailing list
Nagios-users@lists.sourceforge.net
https://lists.sourceforge.net/lists/listinfo/nagios-users
::: Please include Nagios version, plugin version (-v) and OS when reporting 
any issue. 
::: Messages without supporting info will risk being sent to /dev/null


Re: [Nagios-users] Deleting old comments

2011-08-11 Thread Jim Avery
On 11 August 2011 15:17, William Muriithi w.murii...@syncapse.com wrote:
 Jim,

 Why would you want to delete old comments against a host?  I usually like
 them, as it can help identify a recurring problem on the hardware.
 Generally, it's a documentation of the hardware past performance.

 William

Unfortunately performance eventually suffers if there are too many
comments in the system.  I agree, if I could keep them there with
impunity I would.

--
Get a FREE DOWNLOAD! and learn more about uberSVN rich system, 
user administration capabilities and model configuration. Take 
the hassle out of deploying and managing Subversion and the 
tools developers use with it. 
http://p.sf.net/sfu/wandisco-dev2dev
___
Nagios-users mailing list
Nagios-users@lists.sourceforge.net
https://lists.sourceforge.net/lists/listinfo/nagios-users
::: Please include Nagios version, plugin version (-v) and OS when reporting 
any issue. 
::: Messages without supporting info will risk being sent to /dev/null


[Nagios-users] Deleting old comments

2011-08-10 Thread Jim Avery
I'm about half way through writing a perl script to delete old
comments (more than n days old) from Nagios.  Before I spend more time
on it though, if anyone else already has something that does that,
please shout!

--
uberSVN's rich system and user administration capabilities and model 
configuration take the hassle out of deploying and managing Subversion and 
the tools developers use with it. Learn more about uberSVN and get a free 
download at:  http://p.sf.net/sfu/wandisco-dev2dev
___
Nagios-users mailing list
Nagios-users@lists.sourceforge.net
https://lists.sourceforge.net/lists/listinfo/nagios-users
::: Please include Nagios version, plugin version (-v) and OS when reporting 
any issue. 
::: Messages without supporting info will risk being sent to /dev/null


Re: [Nagios-users] Install Error with fresh install of Nagios (nagios.cfg - Line 465)

2011-08-04 Thread Jim Avery
On 4 August 2011 01:56,  matlnx1...@gmail.com wrote:
...
 The line in the nagios file is:

 check_result_path=/usr/local/nagios/var/spool/checkresults
...
 Error:

 Error in configuration file '/usr/local/nagios/etc/nagios.cfg' - Line 465
 (Check result path is not a valid directory)



If it's any help to you, the permissions and ownership of that
directory on my system (which was installed from source tarballs) is:

drwxrwxr-x 2 nagios nagios 507904 2011-08-04 09:29 checkresults

--
BlackBerryreg; DevCon Americas, Oct. 18-20, San Francisco, CA
The must-attend event for mobile developers. Connect with experts. 
Get tools for creating Super Apps. See the latest technologies.
Sessions, hands-on labs, demos  much more. Register early  save!
http://p.sf.net/sfu/rim-blackberry-1
___
Nagios-users mailing list
Nagios-users@lists.sourceforge.net
https://lists.sourceforge.net/lists/listinfo/nagios-users
::: Please include Nagios version, plugin version (-v) and OS when reporting 
any issue. 
::: Messages without supporting info will risk being sent to /dev/null


Re: [Nagios-users] check_procs

2011-07-25 Thread Jim Avery
On 25 July 2011 10:34, Jonathan Aquilina eagles051...@gmail.com wrote:
 is it possible to specify an ip with this check?


No, that plugin is only designed to run locally.


You should be able to run it on the remote machine if you have nrpe
installed though.

See the NRPE manual which you will find here:

http://library.nagios.com/library/products/nagioscore/manuals/


Also/instead, take a look at check_snmp_process which you will find at
http://nagios.manubulon.com/ .  This obviously requires that you have
the monitored system configured to allow SNMP.

If it's a Windows server you want to monitor then take a look at
NSClient++ http://www.nsclient.org/nscp/

I hope that helps!

--
Storage Efficiency Calculator
This modeling tool is based on patent-pending intellectual property that
has been used successfully in hundreds of IBM storage optimization engage-
ments, worldwide.  Store less, Store more with what you own, Move data to 
the right place. Try It Now! http://www.accelacomm.com/jaw/sfnl/114/51427378/
___
Nagios-users mailing list
Nagios-users@lists.sourceforge.net
https://lists.sourceforge.net/lists/listinfo/nagios-users
::: Please include Nagios version, plugin version (-v) and OS when reporting 
any issue. 
::: Messages without supporting info will risk being sent to /dev/null


Re: [Nagios-users] NRPE client issues

2011-07-20 Thread Jim Avery
On 20 July 2011 17:29, Julie S. Lin j...@livescribe.com wrote:
 Hi

 I am monitoring a windows box with very basic out of the box plugins
 using nsclient

 However, whenever we start using this box there is great load on this
 box
 (which is a server class machine)  and causes the nagios checks to time
 out and falsely page.

 Has any one run into this issue and how to resolve?

I come across this sort of thing more so with Unix test servers than Windows.

I expect the easiest way to resolve this would be to configure
NSClient++ to run the checks and send the results to Nagios using
nsca.  That way the agent will run them in its own sweet time and if
they are rather slow it shouldn't matter too much.

Alternatively you might be able to reduce the number of notifications
you get by setting up a servicedependency so all of the checks are
dependent on just one of them.

If the alerts are Unknown rather than Warning or Critical then
you can probably filter out the notifications using the
notification_options directive either on the service definition or
even on the contact definition.

As ever, there are lots of different ways to approach the same
problem!  I hope that helps a little.

--
10 Tips for Better Web Security
Learn 10 ways to better secure your business today. Topics covered include:
Web security, SSL, hacker attacks  Denial of Service (DoS), private keys,
security Microsoft Exchange, secure Instant Messaging, and much more.
http://www.accelacomm.com/jaw/sfnl/114/51426210/
___
Nagios-users mailing list
Nagios-users@lists.sourceforge.net
https://lists.sourceforge.net/lists/listinfo/nagios-users
::: Please include Nagios version, plugin version (-v) and OS when reporting 
any issue. 
::: Messages without supporting info will risk being sent to /dev/null


Re: [Nagios-users] Fwd: parameter starting with dollar sign

2011-07-18 Thread Jim Avery
On 18 July 2011 14:37, Frank Van Damme frank.vanda...@gmail.com wrote:
 I was hoping for some generic escaping rules :)



If only!  I usually end up using a generous portion of trial-and-error
to find something that works.  Sometimes you have to escape the escape
characters that will be used further down the chain.


The most egregious example of multiple escapgin in my configs at the
moment looks like this:

  check_nrpe!-c CheckFile -a MinWarn=0 filter-written=\\8d
file=D:directorysubdirectory\\*.\\*


In another example, I set the $USER10$ macro in resource.cfg to
contain a single semi-colon like so:

  #  Set this macro to contain a semi-colon so we can use it as a
command separator.
  $USER10$=;

And that's just so that it can then be used in an action_url directive
pointing to a DRRAW graph like so:

  action_url
http://nagios/drraw-pnp/index.cgi?Mode=view#59$USER10$Graph=1281540323.16309

Of course the #59 also translates to a semi-colon.  I can't remember
why I have both in there, but it works!

--
AppSumo Presents a FREE Video for the SourceForge Community by Eric 
Ries, the creator of the Lean Startup Methodology on Lean Startup 
Secrets Revealed. This video shows you how to validate your ideas, 
optimize your ideas and identify your business strategy.
http://p.sf.net/sfu/appsumosfdev2dev
___
Nagios-users mailing list
Nagios-users@lists.sourceforge.net
https://lists.sourceforge.net/lists/listinfo/nagios-users
::: Please include Nagios version, plugin version (-v) and OS when reporting 
any issue. 
::: Messages without supporting info will risk being sent to /dev/null


Re: [Nagios-users] Clarification need on configuring nagios to monitor multiple hosts

2011-07-15 Thread Jim Avery
On 14 July 2011 12:14, Zaman quesz...@yahoo.in wrote:
 Is it possible any way to maintain a single entry for all the servers like we
 did for the service object .

You could write a script to create your host definitions based on the
contents of your CMDB (if you have one), or you could configure an
auto-discovery tool to generate the information needed to populate
your host definitions.  I don't think there is anything like that in
Nagios Core itself though, no.

--
AppSumo Presents a FREE Video for the SourceForge Community by Eric 
Ries, the creator of the Lean Startup Methodology on Lean Startup 
Secrets Revealed. This video shows you how to validate your ideas, 
optimize your ideas and identify your business strategy.
http://p.sf.net/sfu/appsumosfdev2dev
___
Nagios-users mailing list
Nagios-users@lists.sourceforge.net
https://lists.sourceforge.net/lists/listinfo/nagios-users
::: Please include Nagios version, plugin version (-v) and OS when reporting 
any issue. 
::: Messages without supporting info will risk being sent to /dev/null


Re: [Nagios-users] Clarification need on configuring nagios to monitor multiple hosts

2011-07-13 Thread Jim Avery
On 13 July 2011 12:24, Zama Ques quesz...@yahoo.in wrote:

 Hi All ,

 I am new to nagios configuration . I  successfully installed nagios on one of 
 my linux server  and now monitoring two hosts .


Welcome!  You've made a good start there.


 For monitoring of remote hosts , I created a new config file by renaming 
 localhost.cfg file under objects directory.

 I renamed the entries hostname and address  and other entries under 
 DEFINE HOST directive to point to the new server which I am planning to 
 monitor . The settings looks like this

 define host{
     use linux-server    ; Name of host 
 template to use
                             ; This host definition will inherit all variables 
 that are defined
                             ; in (or inherited by) the linux-server host 
 template definition.
     host_name   node1
     alias node1
     address       192.168.122.167
     }


 define host{
     use linux-server    ; Name of host 
 template to use
                                    ; This host definition will inherit all 
 variables that are defined
                                    ; in (or inherited by) the linux-server 
 host template definition.
     host_name  node2
     alias        node2
     address  192.168.122.218
     }


Those look fine.


 Also modified DEFINE SERVICE  directive to monitor few services on both the 
 remote hosts . Few of the settings looks like this

 define service{
     use local-service ; Name of 
 service template to use
     host_name   node1
     service_description Current Users
     check_command            check_nrpe!check_users
     }


 define service{
     use local-service ; Name of 
 service template to use
     host_name   node2
     service_description Current Users
     check_command            check_nrpe!check_users
     }

 The checks are working fine . But my confusion here is in terms of scalabilty 
 .
 What if I want to monitor 100 servers and 5 services in each of the servers. 
 Does that mean I need to create 100 define host  directives for each of 
 these servers . Also , monitoring 5 services means i have to create 5 define 
 service for each host , that means 500 define service directives.
 If this is the case , then it will be really painful to configure and manage 
 if there are many servers to monitor.


You'll find useful help on that here:

http://nagios.sourceforge.net/docs/nagioscore/3/en/objecttricks.html




 Can't I use comma separated ip address and hosts like this

 define host{
     use linux-server    ; Name of host 
 template to use
                             ; This host definition will inherit all variables 
 that are defined
                             ; in (or inherited by) the linux-server host 
 template definition.
     host_name   node1,node2
     alias node1
     address       192.168.122.167,192.168.122.218
     }


No.  You do need separate host definitions, but you can add each host
to the same hostgroup for example by adding a directive like this to
the host definition:

  hostgroup_name   linux-group

(in fact you can add that to the host template instead if you like).

You will need to define this new hostgroup:

define hostgroup{
  hostgroup_name   linux-group
  aliaslinux-group hostgroup containing all
monitored linux servers
}



 define service{
     use local-service ; Name of 
 service template to use
     host_name   node1,node2
     service_description Current Users
     check_command            check_nrpe!check_users
     }

In that example, you can now use the hostgroup like so:

 define service{
 use local-service ; Name
of service template to use
 hostgroup_name   linux-group
 service_description Current Users
 check_commandcheck_nrpe!check_users
 }


I hope that helps a little.

--
AppSumo Presents a FREE Video for the SourceForge Community by Eric 
Ries, the creator of the Lean Startup Methodology on Lean Startup 
Secrets Revealed. This video shows you how to validate your ideas, 
optimize your ideas and identify your business strategy.
http://p.sf.net/sfu/appsumosfdev2dev
___
Nagios-users mailing list
Nagios-users@lists.sourceforge.net
https://lists.sourceforge.net/lists/listinfo/nagios-users
::: Please include Nagios 

Re: [Nagios-users] Help: How to use check_snmp plugin to get the monitor APC PDU?

2011-07-12 Thread Jim Avery
On 9 July 2011 15:02, Lee Eric openlinuxsou...@gmail.com wrote:
 Hi,

 Does anyone ever use check_snmp plugin to get the monitor APC PDU? I
 know there are some plugins can do this but I'd like to use this one
 to do more standard. I have downloaded the Powernet MIB file.


An example of how I monitor output percent load is:

check_commandcheck_snmp!-C community -o
UPS-MIB::upsOutputPercentLoad.1 -l \'Output Load\' -u \'%\' -w 10:80
-c 5:90

This is a 3-phase UPS, so I also have checks for the other phases:

check_commandcheck_snmp!-C community -o
UPS-MIB::upsOutputPercentLoad.2 -l \'Output Load\' -u \'%\' -w 10:80
-c 5:90
and
check_commandcheck_snmp!-C community -o
UPS-MIB::upsOutputPercentLoad.3 -l \'Output Load\' -u \'%\' -w 10:80
-c 5:90


The command definition looks like:

define command{
command_namecheck_snmp_ups
command_line$USER1$/check_snmp -H $HOSTADDRESS$ $ARG1$
}



I'd be interested to see what other people do though, particularly as
some of the metrics are *10 the expected value (presumably so they can
be presented as integer rather than floating-point).  I've not been
able to work out the best way of handling those myself yet.

Cheers,

Jim

--
All of the data generated in your IT infrastructure is seriously valuable.
Why? It contains a definitive record of application performance, security 
threats, fraudulent activity, and more. Splunk takes this data and makes 
sense of it. IT sense. And common sense.
http://p.sf.net/sfu/splunk-d2d-c2
___
Nagios-users mailing list
Nagios-users@lists.sourceforge.net
https://lists.sourceforge.net/lists/listinfo/nagios-users
::: Please include Nagios version, plugin version (-v) and OS when reporting 
any issue. 
::: Messages without supporting info will risk being sent to /dev/null


Re: [Nagios-users] Users unable to authenticate, and access the ../nagios site.

2011-07-12 Thread Jim Avery
On 12 July 2011 22:07, dOE doep...@gmail.com wrote:
 Distributor ID: Ubuntu
 Description: Ubuntu 10.10
 Release: 10.10
 Codename: maverick
 Nagios: 3.0.3

 I executed the following commands

 htpasswd -c /usr/local/nagios/etc/htpasswd.users users_name
 service apache2 reload

 I have a local user named user_name as well

 And my cgi.cfg has the usernames as follows:

 use_authentication=1
 authorized_for_system_information=nagiosadmin,users_name
 authorized_for_system_commands=nagiosadmin,users_name
 authorized_for_configuration_information=nagiosadmin,users_name
 authorized_for_all_hosts=nagiosadmin,users_name
 authorized_for_all_host_commands=nagiosadmin,users_name
 authorized_for_all_services=nagiosadmin,users_name
 authorized_for_all_service_commands=nagiosadmin,users_name

 However the user_name can not authenticate with Apache to access the site.

 Does the user need to belong to any particular group other than their own?

Note that users_name is not the same as user_name.

I appreciate your example name might not be the actual name you are
thinking of, but you might need to check you have the spelling spot
on.  You can cat the htpasswd.users file to make sure you have the
name spelled right in there too (it's just a text file).

It might be worth setting the password again using htpasswd
/usr/local/nagios/etc/htpasswd.users users_name just in case the
password was spelled incorrectly the first time you entered it.

Also, check the file /etc/apache2/conf.d/nagios.conf to make sure that
apache is looking for htpasswd.users in the right location.

--
AppSumo Presents a FREE Video for the SourceForge Community by Eric 
Ries, the creator of the Lean Startup Methodology on Lean Startup 
Secrets Revealed. This video shows you how to validate your ideas, 
optimize your ideas and identify your business strategy.
http://p.sf.net/sfu/appsumosfdev2dev
___
Nagios-users mailing list
Nagios-users@lists.sourceforge.net
https://lists.sourceforge.net/lists/listinfo/nagios-users
::: Please include Nagios version, plugin version (-v) and OS when reporting 
any issue. 
::: Messages without supporting info will risk being sent to /dev/null


Re: [Nagios-users] Antwort: Re: PNP4Nagios watermark font colour

2011-07-04 Thread Jim Avery
On 4 July 2011 13:48,  radu.p...@technomatic.de wrote:
 This modifies the entire color font, not only watermark.

Reading the page Joerg mentioned (
http://oss.oetiker.ch/rrdtool/doc/rrdgraph.en.html ) I would guess you
could try the --pango-markup option and format the text in the
watermark using pango markup tags.

The link to the relevant page describing the pango syntax is broken -
I assume it's this one now:
http://developer.gnome.org/pango/stable/PangoMarkupFormat.html

I'll leave it to you to find out whether you have a version of Pango
which supports the required tags (and whether it works at all on the
watermark text or not).

Cheers,

Jim

--
All of the data generated in your IT infrastructure is seriously valuable.
Why? It contains a definitive record of application performance, security 
threats, fraudulent activity, and more. Splunk takes this data and makes 
sense of it. IT sense. And common sense.
http://p.sf.net/sfu/splunk-d2d-c2
___
Nagios-users mailing list
Nagios-users@lists.sourceforge.net
https://lists.sourceforge.net/lists/listinfo/nagios-users
::: Please include Nagios version, plugin version (-v) and OS when reporting 
any issue. 
::: Messages without supporting info will risk being sent to /dev/null


Re: [Nagios-users] Antwort: RE: MRTG graphs problem - measurement units not taken into consideration

2011-07-02 Thread Jim Avery
On 2 July 2011 07:36,  radu.p...@technomatic.de wrote:
 Hello again,

 Can you please give some guidelines how to customize the PNP4Nagios
 background page? I have tried using the $conf['graph_opt'] = --watermark
 'Test' option, but it does not take it. Also modifications done on the
 background.pdf file are not updated. Already restarted apache and nagios
 services. In general I would like to add some logos and/or text.

I can't say I've ever tried that.  I would think you need to append to
rather than replace the graph options though, so wouldn't it be ..

  $conf['graph_opt'] .=  --watermark 'Test'

(note the .= rather than just = and the space added before --watermark )

I don't have my Nagios system to hand at the moment so can't take a
look myself.  Remind me on Monday if you're still stuck and I'll have
a play with that myself.

Another option is to create some graphs outside of PNP altogether (but
using the .rrd databases created by PNP).  See:

http://oss.oetiker.ch/rrdtool/doc/rrdcgi.en.html

Again, I can dig out some examples for you if you remind me during the
week (or do a search - I'm sure we've covered this topic before either
here or on the PNP email list some time in the last few months).

--
All of the data generated in your IT infrastructure is seriously valuable.
Why? It contains a definitive record of application performance, security 
threats, fraudulent activity, and more. Splunk takes this data and makes 
sense of it. IT sense. And common sense.
http://p.sf.net/sfu/splunk-d2d-c2
___
Nagios-users mailing list
Nagios-users@lists.sourceforge.net
https://lists.sourceforge.net/lists/listinfo/nagios-users
::: Please include Nagios version, plugin version (-v) and OS when reporting 
any issue. 
::: Messages without supporting info will risk being sent to /dev/null


Re: [Nagios-users] Whoops! Could not read object configuration data!

2011-07-02 Thread Jim Avery
On 2 July 2011 21:57,  cris...@videotron.ca wrote:
 Hi Community!

 i have a little problem here with Nagios (web interface).

It's not that old chestnut selinux stopping it from working is it?

--
All of the data generated in your IT infrastructure is seriously valuable.
Why? It contains a definitive record of application performance, security 
threats, fraudulent activity, and more. Splunk takes this data and makes 
sense of it. IT sense. And common sense.
http://p.sf.net/sfu/splunk-d2d-c2
___
Nagios-users mailing list
Nagios-users@lists.sourceforge.net
https://lists.sourceforge.net/lists/listinfo/nagios-users
::: Please include Nagios version, plugin version (-v) and OS when reporting 
any issue. 
::: Messages without supporting info will risk being sent to /dev/null


Re: [Nagios-users] nagios-nrpe socket timeout error

2011-07-01 Thread Jim Avery
On 1 July 2011 12:12, rahul kumar rahul@gmail.com wrote:
 Hi,

 I have installed nagios on server and i am monitoring client from this
 machine. Sometimes it shows socket timeout error and next cycle error
 disappear. there is no firewall . All the firewall services are disabled on
 the machine.

Some things to look for ..

- high CPU on the target system

- a mis-match between speed/duplex on the lan interface and
speed/duplex on the switch interface

- a problem with link aggregation (if the system is connected to the
network with more than one interface)

- other network or performance issues

Also look at what the check at the remote end is actually doing.  If
the plugin you are running is particularly slow or inefficient you
will get timeouts.  I have found this particularly with the check_esx3
plugin but also with some other plugins.

Try running your check_nrpe command from the command line using 'time'
a few times to see how it's normally behaving.  You might find that
it's usually taking 8 seconds or so to run and occasionally taking 10
or more seconds.

hth

--
All of the data generated in your IT infrastructure is seriously valuable.
Why? It contains a definitive record of application performance, security 
threats, fraudulent activity, and more. Splunk takes this data and makes 
sense of it. IT sense. And common sense.
http://p.sf.net/sfu/splunk-d2d-c2
___
Nagios-users mailing list
Nagios-users@lists.sourceforge.net
https://lists.sourceforge.net/lists/listinfo/nagios-users
::: Please include Nagios version, plugin version (-v) and OS when reporting 
any issue. 
::: Messages without supporting info will risk being sent to /dev/null


Re: [Nagios-users] Reloading configuration on-the-fly

2011-07-01 Thread Jim Avery
On 1 July 2011 16:22, nag ios nagiost...@gmail.com wrote:
 can u let me know the on-fly process in detail.


You can send a HUP signal to the Nagios daemon to tell it to re-read
the configuration.

Make sure you verify the configs first though:

/usr/local/nagios/bin/nagios -v /usr/local/nagios/etc/nagios.cfg

Then:

kill -1 `cat /usr/local/nagios/var/nagios.lock`

--
All of the data generated in your IT infrastructure is seriously valuable.
Why? It contains a definitive record of application performance, security 
threats, fraudulent activity, and more. Splunk takes this data and makes 
sense of it. IT sense. And common sense.
http://p.sf.net/sfu/splunk-d2d-c2
___
Nagios-users mailing list
Nagios-users@lists.sourceforge.net
https://lists.sourceforge.net/lists/listinfo/nagios-users
::: Please include Nagios version, plugin version (-v) and OS when reporting 
any issue. 
::: Messages without supporting info will risk being sent to /dev/null


Re: [Nagios-users] Reloading configuration on-the-fly

2011-07-01 Thread Jim Avery
On 1 July 2011 20:19, Daniel Wittenberg
daniel.wittenberg.r...@statefarm.com wrote:
 If I remember right though this will only re-read what is there, so if you 
 add/remove files it won't pick those up so we just always use a restart.

I sometimes add/remove a directive
cfg_dir=/usr/local/nagios/etc/objects/test without any problem so I
guess it's behaving better now than it used to.

I do find it's best to do a full stop / start once a week or so
regardless though, as Nagios seems to use increasing amounts of CPU
otherwise.  Mine is quite a busy system though - I need to find some
more beefy hardware to host it!

--
All of the data generated in your IT infrastructure is seriously valuable.
Why? It contains a definitive record of application performance, security 
threats, fraudulent activity, and more. Splunk takes this data and makes 
sense of it. IT sense. And common sense.
http://p.sf.net/sfu/splunk-d2d-c2
___
Nagios-users mailing list
Nagios-users@lists.sourceforge.net
https://lists.sourceforge.net/lists/listinfo/nagios-users
::: Please include Nagios version, plugin version (-v) and OS when reporting 
any issue. 
::: Messages without supporting info will risk being sent to /dev/null


Re: [Nagios-users] Clearing previous flapping states?

2011-06-30 Thread Jim Avery
On 30 June 2011 11:42, Tim Philips t...@rndgroup.co.nz wrote:
 Hi All,

 Tell me, is it possible to pass a command to the Nagios command file to clear 
 (previous) flapping?  I'm picking no - based on my understanding of the 
 documentation but thought I would ask.

 The scenario is we have a number of check software updates that run and as 
 such we schedule these daily.  There is a scenario within the flapping 
 period where there are and aren't updates some of the future warning or 
 critical warnings get suppressed.

 The simple answer that springs to mind for me is to disable flapping for 
 things that are scheduled over x number of hours.  How are other people 
 dealing with such things?

You're right - there is no command you can pass to Nagios to clear the
previous flapping state.  You could try disabling flap detection and
then enabling it again, but my guess is that Nagios will immediately
re-assess the flapping state based on the last 21 checks as soon as
you re-enable flap detection and you'll be back where you started.

If your checks are only run daily, then I would say almost certainly
flap detection is of no benefit to you and should be disabled for
those services.  If Nagios is basing it's assessment of whether the
service is flapping on the last 21 checks as described in the
documentation (
http://nagios.sourceforge.net/docs/nagioscore/3/en/flapping.html ),
then since your checks are daily, it could be a couple of weeks
sometimes before Nagios decides that flapping has stopped!

IMHO, flap detection is only usually useful for those services which
are checked quite frequently.

--
All of the data generated in your IT infrastructure is seriously valuable.
Why? It contains a definitive record of application performance, security 
threats, fraudulent activity, and more. Splunk takes this data and makes 
sense of it. IT sense. And common sense.
http://p.sf.net/sfu/splunk-d2d-c2
___
Nagios-users mailing list
Nagios-users@lists.sourceforge.net
https://lists.sourceforge.net/lists/listinfo/nagios-users
::: Please include Nagios version, plugin version (-v) and OS when reporting 
any issue. 
::: Messages without supporting info will risk being sent to /dev/null


Re: [Nagios-users] How to use check_esx3 Blacklists

2011-06-29 Thread Jim Avery
On 29 June 2011 09:52, Mies, Christian christian.m...@it-novum.com wrote:

 Hi List,

 I try to monitor my Datastores by check_esx3 by Datacenter. I want to exclude 
 each local_* Datastore. The following command is not working ..



 ./check_esx3.pl -D 10.1.82.34 -u user -p pw -l vmfs -x local_*



 What am I doing wrong?


At first glance, my guess is your shell is interpreting the * as a
file wildcard.  You probably need to put quotes around it like so:

./check_esx3.pl -D 10.1.82.34 -u user -p pw -l vmfs -x local_*

Alternatively, backslash-escape it like so:

./check_esx3.pl -D 10.1.82.34 -u user -p pw -l vmfs -x local_\*

It's not clear whether check_esx3.pl supports wildcard notation
though.  Without actually looking at the code I would guess not, or if
it does it might use regular expressions in which case the syntax
would be:

./check_esx3.pl -D 10.1.82.34 -u user -p pw -l vmfs -x local_.*

--
All of the data generated in your IT infrastructure is seriously valuable.
Why? It contains a definitive record of application performance, security 
threats, fraudulent activity, and more. Splunk takes this data and makes 
sense of it. IT sense. And common sense.
http://p.sf.net/sfu/splunk-d2d-c2
___
Nagios-users mailing list
Nagios-users@lists.sourceforge.net
https://lists.sourceforge.net/lists/listinfo/nagios-users
::: Please include Nagios version, plugin version (-v) and OS when reporting 
any issue. 
::: Messages without supporting info will risk being sent to /dev/null


Re: [Nagios-users] Antwort: RE: MRTG graphs problem - measurement units not taken into consideration

2011-06-29 Thread Jim Avery
On 29 June 2011 11:45,  radu.p...@technomatic.de wrote:
 Was a typo error of the community string, after putting public instead of
 Public everything looks cool.

 Still 1 question:

 If I try to query interface GigabitEthernet0/2 I get the response:

 GigabitEthernet0/21:DOWN, GigabitEthernet0/23:DOWN, GigabitEthernet0/24:UP
 No usable data on file (40 rows) , GigabitEthernet0/2:UP No usable data on
 file (1 rows) , GigabitEthernet0/22:DOWN, GigabitEthernet0/20:UP No usable
 data on file (1 rows) : 3 int NOK : CRITICAL

 So ti seems that it takes all interfaces starting with 0/2 string. Any way
 to limit this? Do I have to enter any escape character?

 Thank you!


I think you need the -r option, which means Do not use regexp to
match NAME in description OID.

--
All of the data generated in your IT infrastructure is seriously valuable.
Why? It contains a definitive record of application performance, security 
threats, fraudulent activity, and more. Splunk takes this data and makes 
sense of it. IT sense. And common sense.
http://p.sf.net/sfu/splunk-d2d-c2
___
Nagios-users mailing list
Nagios-users@lists.sourceforge.net
https://lists.sourceforge.net/lists/listinfo/nagios-users
::: Please include Nagios version, plugin version (-v) and OS when reporting 
any issue. 
::: Messages without supporting info will risk being sent to /dev/null


Re: [Nagios-users] Antwort: RE: MRTG graphs problem - measurement units not taken into consideration

2011-06-29 Thread Jim Avery
On 29 June 2011 13:23,  radu.p...@technomatic.de wrote:
 After adding the -r option it works like a charm!

 Now in pnp4nagios I can see both values on the same graph: bytes in and
 bytes out. I see them as two thin lines. Any way to make the lines thicker
 and change their colour?

Well done.

Yes, you can create a custom template.

Take a look under /usr/local/pnp4nagios/share/templates.dist and find
the file named check_snmp_int.php .  Copy it to the directory
/usr/local/pnp4nagios/share/templates and then edit it to get the
wider line.

For example the line which reads:

$def[1] .= LINE1:var1#003300:\in  \  ;

Change it to read:

$def[1] .= LINE3:var1#003300:\in  \  ;



You will find documentation on how the LINE instruction works at:

http://oss.oetiker.ch/rrdtool/doc/rrdgraph_graph.en.html

and some general help on PNP templates at:

http://docs.pnp4nagios.org/pnp-0.6/tpl

--
All of the data generated in your IT infrastructure is seriously valuable.
Why? It contains a definitive record of application performance, security 
threats, fraudulent activity, and more. Splunk takes this data and makes 
sense of it. IT sense. And common sense.
http://p.sf.net/sfu/splunk-d2d-c2
___
Nagios-users mailing list
Nagios-users@lists.sourceforge.net
https://lists.sourceforge.net/lists/listinfo/nagios-users
::: Please include Nagios version, plugin version (-v) and OS when reporting 
any issue. 
::: Messages without supporting info will risk being sent to /dev/null


Re: [Nagios-users] MRTG graphs problem - measurement units not taken into consideration

2011-06-28 Thread Jim Avery
On 28 June 2011 13:31,  radu.p...@technomatic.de wrote:
 Hello,

 I am using Nagios to monitor the bandwidth usage of a couple of
 switches/routers via the check_local_mrtgtraf command. The graphs I handle
 via pnp4nagios. The problem I encounter is the following: if the monitored
 bandwidth is at one point expressed in KB/s and then I bring a lot of
 traffic through the router the, unit is changed into MB/s. The graphed
 values seem not to take into consideration the change of the measurement
 unit meaning that the 50KB/s seems to be at the same height as the 50MB/s
 value in the graph.

 Can anyone please help?


Unless you have already invested a lot of effort in mrtg, I would
recommend using check_snmp_int.pl instead.  You will find it at
http://nagios.manubulon.com/ .

--
All of the data generated in your IT infrastructure is seriously valuable.
Why? It contains a definitive record of application performance, security 
threats, fraudulent activity, and more. Splunk takes this data and makes 
sense of it. IT sense. And common sense.
http://p.sf.net/sfu/splunk-d2d-c2
___
Nagios-users mailing list
Nagios-users@lists.sourceforge.net
https://lists.sourceforge.net/lists/listinfo/nagios-users
::: Please include Nagios version, plugin version (-v) and OS when reporting 
any issue. 
::: Messages without supporting info will risk being sent to /dev/null


Re: [Nagios-users] MRTG graphs problem - measurement units not taken into consideration

2011-06-28 Thread Jim Avery
On 28 June 2011 15:14,  radu.p...@technomatic.de wrote:
 I have tried the SNMP plugin a few minutes ago and here is what I get when
 running it from the command line:

 ./check_snmp_int.pl -H 192.168.2.1 -C public -n GigabitEthernet0/24 -k -w
 200,400 -c 0,600
 GigabitEthernet0/24:UP No usable data on file (1 rows) :(1 UP): UNKNOWN

 The interface name I took it from listing all available interfaces on the
 switch.

 Any ideas, please?

The plugin stores the metrics in a temporary file the first time you
run it.  It will then use that data the next time you run it to
calculate the metrics.  This error shows either that the file did not
exist yet, or the interval between this time the plugin is run and the
last time is wrong.  You therefore need to run the plugin at regular
intervals (say 5 minutes).  It helps if you use SNMP v2 so you can
retrieve 64-bit counters (the -2 command line option and the -g
option).  And you might also need to use the -d option to tell the
plugin at what interval you will usually run it (for example -d 300
for 5 minute intervals).  You need the -f option to tell the plugin
to output performance data.  I normally specify -fY to get the
output in bits/s.

In Nagios you need to set the check_interval and the retry_interval
both to the same 5 minutes (or whatever interval you like so long as
it matches the -d parameter).

Note that if you ran the plugin from the command line as an ordinary
user, it may have created temporary files under /tmp which are not
writeable by Nagios.  When you configure this check to run in Nagios
you may need to delete those temporary files so the nagios user can
write them.

Here is an example of the options I normally use when monitoring
switch interfaces:

-H 198.236.3.4 -C $USER4$ -2 -n Nortel Ethernet Routing Switch
5510-48T Module - Unit 2 Port 5   -fY -kqB -g -w
60,60,100,100,10,10 -c 80,80,200,200,20,20 -d
300

Note that $USER4$ is a macro which I specify in the resources.cfg file
which holds the read-only community string.

You'll find there are a couple of default templates for PNP under the
templates.dist directory - check_snmp_int-bits.php and
check_snmp_int.php - you will of course need your check_command
definition to be named appropriately to pick up whichever of those
templates you need.

hth,

Jim

--
All of the data generated in your IT infrastructure is seriously valuable.
Why? It contains a definitive record of application performance, security 
threats, fraudulent activity, and more. Splunk takes this data and makes 
sense of it. IT sense. And common sense.
http://p.sf.net/sfu/splunk-d2d-c2
___
Nagios-users mailing list
Nagios-users@lists.sourceforge.net
https://lists.sourceforge.net/lists/listinfo/nagios-users
::: Please include Nagios version, plugin version (-v) and OS when reporting 
any issue. 
::: Messages without supporting info will risk being sent to /dev/null


Re: [Nagios-users] Nagios/Sendmail issue

2011-06-28 Thread Jim Avery
On 28 June 2011 18:59, Manish Kumar manikuma...@gmail.com wrote:

 Hi Frnds,

 I have a setup in which nagios 3.2.3 is monitoring a number of servers and 
 network devices like cisco switches and routers. The issue is that i am 
 getting the email notifications very late for any problem.



Nagios will normally send a notification when the service or host goes
from a soft state to a hard state.  When that happens depends mainly
on the max_check_attempts and retry_interval directives.  Only after
the last check attempt fails will Nagios send the notification.  If
you want immediate notification, set max_check_attempts to 1.  Take a
look and if you still can't find the problem, please find the relevant
host definition in your objects.cache file and post it here.

There are various things that can cause Nagios to run slowly.
Important metrics to check are host_check_latency and
service_check_latency - you will find those on the tactical overview
screen.  If either of those are more than 15 seconds or so I would
start investigating why.

Of course you could genuinely have a performance problem in your email
system...  That should be easy to check by sending an email from the
command-line on your Nagios server and checking it arrives reasonably
quickly.

hth,

Jim

--
All of the data generated in your IT infrastructure is seriously valuable.
Why? It contains a definitive record of application performance, security 
threats, fraudulent activity, and more. Splunk takes this data and makes 
sense of it. IT sense. And common sense.
http://p.sf.net/sfu/splunk-d2d-c2
___
Nagios-users mailing list
Nagios-users@lists.sourceforge.net
https://lists.sourceforge.net/lists/listinfo/nagios-users
::: Please include Nagios version, plugin version (-v) and OS when reporting 
any issue. 
::: Messages without supporting info will risk being sent to /dev/null


Re: [Nagios-users] Thanks and configure SENDMAIL for sending mails to other domains

2011-06-25 Thread Jim Avery
On 25 June 2011 12:57, nag ios nagiost...@gmail.com wrote:
 There is no specific reason for choosing sendmail , if you can provide me
 information for configuring through postfix i will accept it..

How you configure postfix depends a lot on what linux distro you are
using.  Each one I've tried packages it differently, but all have been
quite easy to configure.  A quick google query for postfix
configuration for whatever distro you are on will probably get you
some instructions.

If you're using either Mandriva or Ubuntu I can probably point you in
the right direction.

--
All the data continuously generated in your IT infrastructure contains a 
definitive record of customers, application performance, security 
threats, fraudulent activity and more. Splunk takes this data and makes 
sense of it. Business sense. IT sense. Common sense.. 
http://p.sf.net/sfu/splunk-d2d-c1
___
Nagios-users mailing list
Nagios-users@lists.sourceforge.net
https://lists.sourceforge.net/lists/listinfo/nagios-users
::: Please include Nagios version, plugin version (-v) and OS when reporting 
any issue. 
::: Messages without supporting info will risk being sent to /dev/null


Re: [Nagios-users] Thanks and configure SENDMAIL for sending mails to other domains

2011-06-24 Thread Jim Avery
On 24 June 2011 12:16, nag ios nagiost...@gmail.com wrote:
 Thanks for all of you for helping me to configure Nagios.It's running well

     But, i need one more thing.I am thinking to configure sendmail
 such way that if any service is down or warning , mail should be delivered
 to some specific person stating the state of the service.For this i need to
 configure SENDMAIL first for sending mails to other domains can you help.


Is there any particular reason you want to use sendmail?  Personally I
find postfix a lot easier to install and configure (on Ubuntu and
Mandriva versions of Linux - I've not tried RedHat or Suse, etc,.).

--
All the data continuously generated in your IT infrastructure contains a 
definitive record of customers, application performance, security 
threats, fraudulent activity and more. Splunk takes this data and makes 
sense of it. Business sense. IT sense. Common sense.. 
http://p.sf.net/sfu/splunk-d2d-c1
___
Nagios-users mailing list
Nagios-users@lists.sourceforge.net
https://lists.sourceforge.net/lists/listinfo/nagios-users
::: Please include Nagios version, plugin version (-v) and OS when reporting 
any issue. 
::: Messages without supporting info will risk being sent to /dev/null


Re: [Nagios-users] More SNMP CONF files

2011-06-22 Thread Jim Avery
On 22 June 2011 03:59, Chris Schaft ch...@mkadvantage.com wrote:

 We have posted several hundred more conf files for trap handling.  You can 
 get them
 at http://www.mkadvantage.com.  Once you get here, go to Tools on the Menu 
 Bar,
 then go to SNMP Trap Conf files on the left side.


That looks extremely useful - it can be a right pain tracking down the
relevant MIB file otherwise.  Many thanks!

Jim

--
Simplify data backup and recovery for your virtual environment with vRanger.
Installation's a snap, and flexible recovery options mean your data is safe,
secure and there when you need it. Data protection magic?
Nope - It's vRanger. Get your free trial download today.
http://p.sf.net/sfu/quest-sfdev2dev
___
Nagios-users mailing list
Nagios-users@lists.sourceforge.net
https://lists.sourceforge.net/lists/listinfo/nagios-users
::: Please include Nagios version, plugin version (-v) and OS when reporting 
any issue. 
::: Messages without supporting info will risk being sent to /dev/null


Re: [Nagios-users] Scheduled downtime and host checks

2011-05-31 Thread Jim Avery
On 31 May 2011 17:18, Kumar, Ashish xml.de...@gmail.com wrote:
 Hello Everyone,

 I had been under the impression that no service checks would be executed on
 a server during scheduled downtime but it seems otherwise.

 Would I be right in assuming all service checks will still be performed on a
 server during a scheduled downtime?

 I tried checking the documentation but there seems to be no information
 regarding this.

 Any hints are most welcome.

No, scheduled downtime only affects notifications, and the stats you
see in the availability cgi.  Service and host checks run as normal
during scheduled downtime.  This is useful because it allows you to
check the stats of those hosts and services are ok before the
scheduled downtime period ends.

http://nagios.sourceforge.net/docs/nagioscore/3/en/downtime.html

--
Simplify data backup and recovery for your virtual environment with vRanger. 
Installation's a snap, and flexible recovery options mean your data is safe,
secure and there when you need it. Data protection magic?
Nope - It's vRanger. Get your free trial download today. 
http://p.sf.net/sfu/quest-sfdev2dev
___
Nagios-users mailing list
Nagios-users@lists.sourceforge.net
https://lists.sourceforge.net/lists/listinfo/nagios-users
::: Please include Nagios version, plugin version (-v) and OS when reporting 
any issue. 
::: Messages without supporting info will risk being sent to /dev/null


Re: [Nagios-users] Passive check windows clients

2011-05-17 Thread Jim Avery
On 17 May 2011 10:31, Andre Tann at...@alphasrv.net wrote:


Don't use a mac personally, but you can schedule the check locally at a
frequency you desire, and submit this to a passive job def with its
freshness
check disabled so it will never timeout.

 Ok. And what piece of software would be needed on the windows side to check 
 disk/load?

 Thanks,
 Andre

The NSClient++ agent will do this quite nicely.  You will of course
need nsca installed on your Nagios server to receive and process the
incoming alerts.

The config directives in the NSClient++ configuration file will need
to look something like this:


[modules]
NSCAAgent.dll

[NSCA Agent]
;# CHECK INTERVAL (in seconds)
;   How often we should run the checks and submit the results.
interval=300
;
;# ENCRYPTION METHOD
;   This option determines the method by which the send_nsca client
will encrypt the packets it sends
;   to the nsca daemon. The encryption method you choose will be a
balance between security and
;   performance, as strong encryption methods consume more processor resources.
;   You should evaluate your security needs when choosing an encryption method.
;
; Note: The encryption method you specify here must match the
decryption method the nsca daemon uses
;   (as specified in the nsca.cfg file)!!
; Values:
;   0 = None(Do NOT use this option)
;   1 = Simple XOR  (No security, just obfuscation, but very fast)
;   2 = DES
;   3 = 3DES (Triple DES)
;   4 = CAST-128
;   6 = xTEA
;   8 = BLOWFISH
;   9 = TWOFISH
;   11 = RC2
;   14 = RIJNDAEL-128 (AES)
;   20 = SERPENT
encryption_method=1
;
;# ENCRYPTION PASSWORD
;  This is the password/passphrase that should be used to encrypt the
sent packets.
password=obfuscate
;
;# BIND TO ADDRESS
;  Allows you to bind server to a specific local address. This has to
be a dotted ip adress not a hostname.
;  Leaving this blank will bind to one local interface.
; -- not supported as of now --
;bind_to_address=
;
;# LOCAL HOST NAME
;  The name of this host (if empty computername will be used.
;hostname=
;
;# NAGIOS SERVER ADDRESS
;  The address to the nagios server to submit results to.
nsca_host=100.1.2.3
;
;# NAGIOS SERVER PORT
;  The port to the nagios server to submit results to.
nsca_port=5667
;


[NSCA Commands]

CPU=CheckCPU warn=98 crit=99 time=1m warn=90 crit=95 time=5m warn=80
crit=90 time=15m ShowAll
Disks=CheckDriveSize ShowAll MaxWarnUsed=90% MaxCritUsed=95%
CheckAllOthers FilterType=FIXED Drive=E:\



You will probably want to include a passive host check too - I haven't
configured one here as I use an active check myself for those.


hth,

Jim

--
Achieve unprecedented app performance and reliability
What every C/C++ and Fortran developer should know.
Learn how Intel has extended the reach of its next-generation tools
to help boost performance applications - inlcuding clusters.
http://p.sf.net/sfu/intel-dev2devmay
___
Nagios-users mailing list
Nagios-users@lists.sourceforge.net
https://lists.sourceforge.net/lists/listinfo/nagios-users
::: Please include Nagios version, plugin version (-v) and OS when reporting 
any issue. 
::: Messages without supporting info will risk being sent to /dev/null


Re: [Nagios-users] RFC/RFP: Service parents

2011-05-17 Thread Jim Avery
On 17 May 2011 09:57, Andreas Ericsson a...@op5.se wrote:

 Comments and patches welcome.

That sounds good - servicedependencies can be quite painful to understand.

Ultimately I would quite like it if the cgi could display the child
services indented below the parent to make it more obvious what the
relationships are to the end user.  Likewise, I would like the option,
much like the 'blocking outages' screen to be able to view only the
blocking service, not the affected child services.

Cheers,

Jim

--
Achieve unprecedented app performance and reliability
What every C/C++ and Fortran developer should know.
Learn how Intel has extended the reach of its next-generation tools
to help boost performance applications - inlcuding clusters.
http://p.sf.net/sfu/intel-dev2devmay
___
Nagios-users mailing list
Nagios-users@lists.sourceforge.net
https://lists.sourceforge.net/lists/listinfo/nagios-users
::: Please include Nagios version, plugin version (-v) and OS when reporting 
any issue. 
::: Messages without supporting info will risk being sent to /dev/null


Re: [Nagios-users] Newbie Question

2011-05-16 Thread Jim Avery
On 16 May 2011 14:47, Matty Sarro msa...@gmail.com wrote:
 This is for mysql. I saw a plugin called check_mysql_query but it
 doesn't seem to be included in the plugins package I downloaded. I'm
 going to try reinstalling the plugins in case something got borked. I
 can't find it on exchange either.

 I may have one other possibility after some more searching:
 check_mysql_count

 Any further suggestions would really be appreciated though.


If you can't find anything suitable 'off the shelf' then there is a
plugin check_snmptraps.pl included with the NagTrap addon which is
supposed to be for checking the number of traps in the nagtrap
database but I expect it would be easy to rewrite it to query whatever
MySQL database you like.

--
Achieve unprecedented app performance and reliability
What every C/C++ and Fortran developer should know.
Learn how Intel has extended the reach of its next-generation tools
to help boost performance applications - inlcuding clusters.
http://p.sf.net/sfu/intel-dev2devmay
___
Nagios-users mailing list
Nagios-users@lists.sourceforge.net
https://lists.sourceforge.net/lists/listinfo/nagios-users
::: Please include Nagios version, plugin version (-v) and OS when reporting 
any issue. 
::: Messages without supporting info will risk being sent to /dev/null


Re: [Nagios-users] Notification after Acknowledgment

2011-05-13 Thread Jim Avery
On 13 May 2011 08:34, Andre Kruger andre.kru...@trw.com wrote:
 Hi

 Can you guys please give me your input on how you handle the following
 situation.

 Lets take monitoring a disk as an example. For arguments sake lets say when
 the disk reaches 80% capacity I send out a warning and at 90% I send out a
 critical. There is also a Service Escalation configured to send out
 notifications when this service reaches critical.

 So at 80 percent I get my notification all is well. I then go ahead and
 acknowledge the event and in doing so Nagios will not send out any further
 notifications. Which according to the Nagios logic is correct.

 The problem is if the disk in the mean time reaches critical, 90% capacity,
 I won't get another notification. Not even the Service Escalation helps
 here, because the event has already been acknowledged.

 Do you guys have any suggestions on how this problem can be solved?

 Regards
 Andre


The way I sometimes use for prolonged issues like this is I will
acknowledge the alert, but then raise the warning and critical
thresholds in Nagios.  The problem with this approach is that Nagios
then reports the status as OK which might give a false impression to
other users.  It is also important to remember to reduce the warning
threshold back to its usual level once the issue is resolved.

For issues which might be fast-moving I would suggest that it is not
appropriate to acknowledge the issue unless you are in a postion
actively to manage it until resolution.

--
Achieve unprecedented app performance and reliability
What every C/C++ and Fortran developer should know.
Learn how Intel has extended the reach of its next-generation tools
to help boost performance applications - inlcuding clusters.
http://p.sf.net/sfu/intel-dev2devmay
___
Nagios-users mailing list
Nagios-users@lists.sourceforge.net
https://lists.sourceforge.net/lists/listinfo/nagios-users
::: Please include Nagios version, plugin version (-v) and OS when reporting 
any issue. 
::: Messages without supporting info will risk being sent to /dev/null


Re: [Nagios-users] Notification after Acknowledgment

2011-05-13 Thread Jim Avery
On 13 May 2011 09:01, Andre Kruger andre.kru...@trw.com wrote:

 I just read how non-sticky acknowledgments work from 3.2.3. I think this
 solves my problem.

 http://wiki.nagios.org/index.php/Acknowledgementlogic


Neat!  Thanks I hadn't noticed that.

--
Achieve unprecedented app performance and reliability
What every C/C++ and Fortran developer should know.
Learn how Intel has extended the reach of its next-generation tools
to help boost performance applications - inlcuding clusters.
http://p.sf.net/sfu/intel-dev2devmay
___
Nagios-users mailing list
Nagios-users@lists.sourceforge.net
https://lists.sourceforge.net/lists/listinfo/nagios-users
::: Please include Nagios version, plugin version (-v) and OS when reporting 
any issue. 
::: Messages without supporting info will risk being sent to /dev/null


Re: [Nagios-users] Message Subject - PROBLEM Service Alert: hostname is UP

2011-05-13 Thread Jim Avery
On 13 May 2011 16:09, Terry Carmen te...@cnysupport.com wrote:
 I've been receiving a continuous stream of these messages and can't
 seem to figure out why or make them stop.

 Nagios is sending repeated notifications that various hosts are up
 even though they have never been down.

 I've googled a number of similar problems, but unfortunately no
 applicable solutions.

 Does anybody have any ideas what I could check?

I would find the relevant host entry in the file
/usr/local/nagios/var/objects.cache, then make sure you understand all
the directives in that host definition - by all means post it here too
if you want a group opinion.

Check that you don't have more than one contact defined having the
same email address.

Also, check the status history for that host and see if anything is
happening in-between those times when it is UP.  Maybe it's going DOWN
in-between but you're just not getting the DOWN alerts!

It would be useful to know a bit more about what you're seeing, for
example are these repeat emails occurring with a particular frequency?
  Do you get a whole bunch of them at once and then none for a while?
Do they eventually stop and if so after how many repeats?

hth,

Jim

--
Achieve unprecedented app performance and reliability
What every C/C++ and Fortran developer should know.
Learn how Intel has extended the reach of its next-generation tools
to help boost performance applications - inlcuding clusters.
http://p.sf.net/sfu/intel-dev2devmay
___
Nagios-users mailing list
Nagios-users@lists.sourceforge.net
https://lists.sourceforge.net/lists/listinfo/nagios-users
::: Please include Nagios version, plugin version (-v) and OS when reporting 
any issue. 
::: Messages without supporting info will risk being sent to /dev/null


Re: [Nagios-users] Nagios + Jasper

2011-05-11 Thread Jim Avery
On 11 May 2011 12:22, trm asn trm.nag...@gmail.com wrote:
 Hi List,

 Has anyone configured Nagios with Jasper Report or anyone having any better
 solutions with existing nagios(3.2).


I haven't tried it myself at all, but (in no particular order and
according to their web sites listed below) ...

it looks like Netways have:

http://www.netways.de/en/de/produkte/jasper/integration/

and it's used in Opsera's reports module:

http://www.opsview.com/sites/default/files/Opsview%20Reports%20Module_0.pdf

and OpsView uses it too:

http://opsview.com/products/opsview-enterprise/reporting-dashboards


hth,

Jim

--
Achieve unprecedented app performance and reliability
What every C/C++ and Fortran developer should know.
Learn how Intel has extended the reach of its next-generation tools
to help boost performance applications - inlcuding clusters.
http://p.sf.net/sfu/intel-dev2devmay
___
Nagios-users mailing list
Nagios-users@lists.sourceforge.net
https://lists.sourceforge.net/lists/listinfo/nagios-users
::: Please include Nagios version, plugin version (-v) and OS when reporting 
any issue. 
::: Messages without supporting info will risk being sent to /dev/null


Re: [Nagios-users] Before I RTFM....

2011-05-03 Thread Jim Avery
On 3 May 2011 19:43, Dj Lien dj_l...@hotmail.com wrote:

 .. does anyone have any links/pointers for getting Nagios to process SNMP
 traps.

snmptt is what you need.

I've written some stuff about what you can do here:

http://wiki.nagios.org/index.php/Howtos:snmp-windows

You might also/instead like to look at a tool called NagTrap.

Shout again if you get stuck.

hth,

Jim

--
WhatsUp Gold - Download Free Network Management Software
The most intuitive, comprehensive, and cost-effective network 
management toolset available today.  Delivers lowest initial 
acquisition cost and overall TCO of any competing solution.
http://p.sf.net/sfu/whatsupgold-sd
___
Nagios-users mailing list
Nagios-users@lists.sourceforge.net
https://lists.sourceforge.net/lists/listinfo/nagios-users
::: Please include Nagios version, plugin version (-v) and OS when reporting 
any issue. 
::: Messages without supporting info will risk being sent to /dev/null


Re: [Nagios-users] Freshness Checking not working correctly

2011-04-21 Thread Jim Avery
On 21 April 2011 01:51, Samuel Kidman samuel.kid...@panres.com wrote:
 Hello

 I've got this in my logs:


 [1303346647] Warning: The results of service 'Dummy Service' on host
 'LFMISS1' are stale by 0d 0h 1m 0s (threshold=0d 0h 5m 0s).  I'm forcing
 an immediate check of the service.
 [1303346647] Warning: The results of service 'Dummy Service' on host
 'PR-LNM-BCMF01' are stale by 0d 0h 1m 0s (threshold=0d 0h 5m 0s).  I'm
 forcing an immediate check of the service.
 [1303346647] Warning: The results of service 'Dummy Service' on host
 'PR-LNM-BW-AV01' are stale by 0d 0h 1m 0s (threshold=0d 0h 5m 0s).  I'm
 forcing an immediate check of the service.


 Why  is Nagios forcing an active check before the freshness threshold
 runs out?



I've been seeing something that looks similar to that.

An example in my log is:

[1303254000] CURRENT SERVICE STATE:
xx;BACKUP;OK;HARD;1;backup_app: Completed with no errors.
...
[1303284031] Warning: The results of service 'BACKUP' on host 'xx'
are stale by 0d 0h 0m 52s (threshold=1d 2h 0m 0s).  I'm forcing an
immediate check of the service.

In my case, it seems Nagios thought the service was stale only 8 hours
or so after the previous passive check, whereas the freshness
threshold for this service is 93600 (26 hours).  I'm not sure why,
then, the warning message says it's stale by only 52s.


One thing I have done recently is to move the status.dat file from its
usual location to a ramdisk.  This has improved performance in the web
front end, but the web front end occasionally loses sight of the
services.  I wonder if the freshness checking also reads status.dat
and is having similar problems?  I expect I might have to back out the
change and move status.dat back to its original location on the ext3
filesystem.  Out of interest, have you moved any of your Nagios files
to a ramdisk?

I notice the Nagios documentation no longer makes any mention of
moving files to ramdisk to improve performance (if it ever did - I
can't seem to find the Nagios 2 documentation online any more).  If
anyone can shed any light on this I'd appreciate it.

For the record I'm using Nagios Core 3.2.3 on Ubuntu 08.04.01 LTS
Server, installed from tarballs, not packages.

Cheers,

Jim

--
Benefiting from Server Virtualization: Beyond Initial Workload 
Consolidation -- Increasing the use of server virtualization is a top
priority.Virtualization can reduce costs, simplify management, and improve 
application availability and disaster protection. Learn more about boosting 
the value of server virtualization. http://p.sf.net/sfu/vmware-sfdev2dev
___
Nagios-users mailing list
Nagios-users@lists.sourceforge.net
https://lists.sourceforge.net/lists/listinfo/nagios-users
::: Please include Nagios version, plugin version (-v) and OS when reporting 
any issue. 
::: Messages without supporting info will risk being sent to /dev/null


Re: [Nagios-users] Monitoring Bandwidth Question

2011-04-19 Thread Jim Avery
On 19 April 2011 00:33, David Barszcz dbars...@gmail.com wrote:
 Hello,

 I need help monitoring bandwidth of my router. I am currently running nagios
 on Ubuntu and monitoring a router flashed with DD-WRT. My plan is to monitor
 the bandwidth throught MRTG. I seem to be having a problems creating a log
 file of router's Bandwidth to point to. This should be an easy one so a
 Litle guidance would appreciated.

If the router can accept SNMP queries then you should be able to use
http://nagios.manubulon.com/snmp_int.html and rather than use MRTG I
would recommend PNP4Nagios as this can get the performance data from
Nagios.

Note that check_snmp_int.pl will write temporary files to keep a
record of counters so be careful of ownership and permissions when
testing.   Also, you need to specify the 64-bit counters otherwise
they wrap round back to zero too frequently.

hth,

Jim

--
Benefiting from Server Virtualization: Beyond Initial Workload 
Consolidation -- Increasing the use of server virtualization is a top
priority.Virtualization can reduce costs, simplify management, and improve 
application availability and disaster protection. Learn more about boosting 
the value of server virtualization. http://p.sf.net/sfu/vmware-sfdev2dev
___
Nagios-users mailing list
Nagios-users@lists.sourceforge.net
https://lists.sourceforge.net/lists/listinfo/nagios-users
::: Please include Nagios version, plugin version (-v) and OS when reporting 
any issue. 
::: Messages without supporting info will risk being sent to /dev/null


Re: [Nagios-users] Running configuration: Additional macros or sourcing files for passwords

2011-04-19 Thread Jim Avery
This is standard stuff in Nagios.  Usually the password is stored in a
macro which you define in the resource.cfg file, for example:

$USER3$=trustnoone

Then you can refer to the macro name in your service or command
definitions for example:

define service{
  use   srv-pnp,generic-service
  host_name coreswitch1
  service_description   Unit04-Port18
  check_command check_snmp_int!-C $USER3$ -2 -n Unit 4 Port
18   -fY -kqB -g -w 60,60,100,100,10,10 -c
80,80,200,200,20,20 -d 300
}

Permissions are set appropriately on the resource.cfg file, and the
resource.cfg file doesn't have to be in your code repo.

Reference is made to this in the documentation at
http://nagios.sourceforge.net/docs/nagioscore/3/en/configmain.html
under the heading Resource File - you should also find examples in
the sample config files which come with the standard Nagios install.

hth,

Jim


On 18 April 2011 23:53, d icantplaygui...@gmail.com wrote:
 Looking for info on how passwords are handled in configuration files.  I
 would preferably source them from a hidden file and then insert them via
 macros into the nagios config.  Has anyone done this (or similar)?  I like
 having my configuration files checked-in to a code repo but obviously don't
 want the PWs.
 Thanks.
 --
 Benefiting from Server Virtualization: Beyond Initial Workload
 Consolidation -- Increasing the use of server virtualization is a top
 priority.Virtualization can reduce costs, simplify management, and improve
 application availability and disaster protection. Learn more about boosting
 the value of server virtualization. http://p.sf.net/sfu/vmware-sfdev2dev
 ___
 Nagios-users mailing list
 Nagios-users@lists.sourceforge.net
 https://lists.sourceforge.net/lists/listinfo/nagios-users
 ::: Please include Nagios version, plugin version (-v) and OS when reporting
 any issue.
 ::: Messages without supporting info will risk being sent to /dev/null


--
Benefiting from Server Virtualization: Beyond Initial Workload 
Consolidation -- Increasing the use of server virtualization is a top
priority.Virtualization can reduce costs, simplify management, and improve 
application availability and disaster protection. Learn more about boosting 
the value of server virtualization. http://p.sf.net/sfu/vmware-sfdev2dev
___
Nagios-users mailing list
Nagios-users@lists.sourceforge.net
https://lists.sourceforge.net/lists/listinfo/nagios-users
::: Please include Nagios version, plugin version (-v) and OS when reporting 
any issue. 
::: Messages without supporting info will risk being sent to /dev/null


Re: [Nagios-users] backup email notifcation consolidator

2011-04-13 Thread Jim Avery
On 13 April 2011 00:41, Matthew Nickerson mnicker...@silcon.com wrote:
 Hello,



 I’m looking for an add-on, and hoping someone has made it, that can
 consolidate backup email reports.  We have several clients, running multiple
 version of backup exec, some with NTbackup, and some with Jungle Disc.  I’d
 like to get all these email reports consolidated into a single report that
 shows if a backup ran when it was supposed to run, and if the job was
 successful.



 Alternatively, if Nagios can monitor an inbox and parse the emails for
 failure messages and generate a warning, that would work too.



 Right now the problem is that I get about 60 reports daily, which clutters
 up the inbox.


I wouldn't know about Jungle Disc, but I can tell you that Backup Exec
can (and probably should) be configured to send an SNMP trap showing
backup status.  I would install and configure snmptt to receive these
traps and send passive check results to Nagios.  Nagios will then only
email you for those backups that have failed not those that have
succeeded.  Note you should also use freshness checking in the Nagios
passive service check config to alert if no check result at all has
been received for a given period.

I've not tried it myself, but for log file consolidation, I believe
Splunk is the recommended tool.

http://www.nagios.org/products/enterprisesolutions/splunk/

--
Forrester Wave Report - Recovery time is now measured in hours and minutes
not days. Key insights are discussed in the 2010 Forrester Wave Report as
part of an in-depth evaluation of disaster recovery service providers.
Forrester found the best-in-class provider in terms of services and vision.
Read this report now!  http://p.sf.net/sfu/ibm-webcastpromo
___
Nagios-users mailing list
Nagios-users@lists.sourceforge.net
https://lists.sourceforge.net/lists/listinfo/nagios-users
::: Please include Nagios version, plugin version (-v) and OS when reporting 
any issue. 
::: Messages without supporting info will risk being sent to /dev/null


Re: [Nagios-users] Trend Micro Officescan snmp-trap

2011-04-06 Thread Jim Avery
I'm not sure the specifics of getting Trend to send traps (I simply
asked my Trend admin person to set it up!).  Trend should be able to
provide you with the relevant MIB file.

To configure Nagios to receive the traps, I recommend you use snmptt.
The specific instructions relating to Nagios are at:

http://www.snmptt.org/docs/snmptt.shtml#Nagios-Netsaint


You will need to use the snmpttconvertmib utility to convert Trend's
MIB file to a config for snmptt.  Note I found that the format of the
OID's for the traps generated from our Trend systems didn't always
match what was in the MIB, so I had to create some new config entries
for snmptt based on the ones translated using snmpttconvertmib, but
with the OIDs edited to match what we were receiving - for example:


# These traps are pretty much same as from trend.mib, but with .999.
instead of .141.
EVENT virusFound .1.3.6.1.4.1.6101.999.3.3 Status Events critical
FORMAT Virus Found Trap: $*
SDESC
 This event trap will be sent when a virus is found
Variables:
  1: tvcsEventTrapVar
PREEXEC /bin/echo $s | /bin/sed -e s/ok/0/ -e s/warning/1/ -e s/critical/2/
EXEC /usr/local/nagios/libexec/eventhandlers/submit_check_result $A
SNMP-Trap-Trend $p1 $N $*
EDESC
#
#
#


Note the PREEXEC and EXEC entries which are used to translate the
incoming trap in to a format which can be submitted as a passive check
to Nagios via Nagios' command interface.  You may need to edit the
EVENT line in each config entry to change the severity as appropriate
to ok, warning, or critical so it can be processed by the
PREEXEC line.


I hope that helps a little.

Cheers,

Jim

--
Xperia(TM) PLAY
It's a major breakthrough. An authentic gaming
smartphone on the nation's most reliable network.
And it wants your games.
http://p.sf.net/sfu/verizon-sfdev
___
Nagios-users mailing list
Nagios-users@lists.sourceforge.net
https://lists.sourceforge.net/lists/listinfo/nagios-users
::: Please include Nagios version, plugin version (-v) and OS when reporting 
any issue. 
::: Messages without supporting info will risk being sent to /dev/null


Re: [Nagios-users] Donwtime_sched not working correctly

2011-03-31 Thread Jim Avery
On 23 March 2011 13:12, tom sturme tom.stu...@live.nl wrote:


 
 From: tom.stu...@live.nl
 To: nagios-users-requ...@lists.sourceforge.net
 Subject: Donwtime_sched not working correctly
 Date: Wed, 23 Mar 2011 13:00:34 +0100

 Hi,

 I'm trying to run downtime_sched 3.2 with nagios 3.2.2


That's interesting.  I'm on version .30 myself.  I must upgrade sometime!


 I've done the steps that are defined in the readme file.


I can't remember why, but I have the job running at 1 minute past 1
each morning in cron like so:

$ crontab -u nagios -l
  # m h  dom mon dow   command
  # Automated scheduled down-time for Nagios.
  01 01 * * * /usr/local/nagios/bin/downtime_job.pl 
/usr/local/nagios/var/downtime.log 21

To be honest it's probably better if it runs later as you have it as
it might not run when the clocks change each year the way I have it.

Note that unlike the instructions in the README, I have it outputting
to a log file.  This is really helpful in seeing if the cron job is
working or not.  Give it a try, see what comes up in the log file the
next morning and if you're still stuck I'll take a closer look for
you.

Cheers,

Jim

--
Create and publish websites with WebMatrix
Use the most popular FREE web apps or write code yourself; 
WebMatrix provides all the features you need to develop and 
publish your website. http://p.sf.net/sfu/ms-webmatrix-sf
___
Nagios-users mailing list
Nagios-users@lists.sourceforge.net
https://lists.sourceforge.net/lists/listinfo/nagios-users
::: Please include Nagios version, plugin version (-v) and OS when reporting 
any issue. 
::: Messages without supporting info will risk being sent to /dev/null


Re: [Nagios-users] Donwtime_sched not working correctly

2011-03-31 Thread Jim Avery
p.s. I see in your email you are already outputting to a log file.  Is
there anything interesting in your /usr/local/nagios/logs/downtime.log
file?

I see you have yours in the system wide crontab.  I don't know if it
makes a huge difference but I guess it might help to have it in the
crontab for user nagios instead.

Cheers,

Jim

--
Create and publish websites with WebMatrix
Use the most popular FREE web apps or write code yourself; 
WebMatrix provides all the features you need to develop and 
publish your website. http://p.sf.net/sfu/ms-webmatrix-sf
___
Nagios-users mailing list
Nagios-users@lists.sourceforge.net
https://lists.sourceforge.net/lists/listinfo/nagios-users
::: Please include Nagios version, plugin version (-v) and OS when reporting 
any issue. 
::: Messages without supporting info will risk being sent to /dev/null


Re: [Nagios-users] Question on encryption for send_nsca

2011-03-31 Thread Jim Avery
On 23 March 2011 23:51, Steve F steve_...@hotmail.com wrote:
 I have a small test Nagios 3.2 environment on a Suse platform ( 11  10
 boxes )and was wondering about the encryption methods used.

 I am working fine with no encryption configured on host or central server. I
 am getting all of the checks on the console.  Once I set it to any value
 other than 0 on both the server  host, I no longer get anything on the
 central server.  I've tried using the simple XOR and 3DES  DES

 If I send a test job from the command line as per the instructions, with no
 encryption set ( not even simple XOR ) I see the reply in the nagios.log
 file on the Nagios server.  If I set the encryption on both  send the test
 job again,( restarting nagios on both)  it says 1 packet was sent
 successfully but nothing shows up on the central server.

 I am still learning my way thru Suse  Nagios so not sure if the encryption
 process is part of the send_nsca or Suse.  I don't know what encryption is
 installed on the suse boxes or even how to check.

 Is there a way to test or check what encryption(s) are installed on the
 boxes?

 Since this is ultimately a totally private network, can I use the value of 0
 for no encryption ?  I know the cfg file explicitly says its not recommended


I was using LOKI97 (can't remember why now) for Unix systems but then
found I couldn't get it working for Windows systems and ended up
configuring a separate nsca daemon to receive checks from Windows
servers with XOR encryption.

Personally unless you're exposed to the outside world or have any
checks which transfer passwords or other sensitive information over
nsca I wouldn't worry about encryption - it's probably more hassle
than it's worth.

hth,

Jim

--
Create and publish websites with WebMatrix
Use the most popular FREE web apps or write code yourself; 
WebMatrix provides all the features you need to develop and 
publish your website. http://p.sf.net/sfu/ms-webmatrix-sf
___
Nagios-users mailing list
Nagios-users@lists.sourceforge.net
https://lists.sourceforge.net/lists/listinfo/nagios-users
::: Please include Nagios version, plugin version (-v) and OS when reporting 
any issue. 
::: Messages without supporting info will risk being sent to /dev/null


Re: [Nagios-users] nagios send_nsca

2011-03-31 Thread Jim Avery
On 31 March 2011 14:42,  julian_grunn...@tdwh.co.uk wrote:

 Hi - hope someone can help me with the above, I want to use this to send
 multiple lines of output via send_nsca and them to be displayed as separate
 lines in Nagios under the Status Information box BUT ALL for the same
 Service check. I've done this in the past and can't for the life of me
 remember how I did it now. So the input file will be something like this:

 server    service check status code       This is line 1br This is
 line 2br This is line 3br etc.

 Like I say I'm sure at a previous role I was doing it like this and it
 worked fine, however now all I get displayed in Nagios is:

 This is line 1br This is line 2br This is line 3br etc.

 Rather than:

 This is line 1
 This is line 2
 This is line 3
 etc.


I'm not entirely sure, but is it this directive in cgi.cfg you need?:-

# ESCAPE HTML TAGS
# This option determines whether HTML tags in host and service
# status output is escaped in the web interface.  If enabled,
# your plugin output will not be able to contain clickable links.

escape_html_tags=0

--
Create and publish websites with WebMatrix
Use the most popular FREE web apps or write code yourself; 
WebMatrix provides all the features you need to develop and 
publish your website. http://p.sf.net/sfu/ms-webmatrix-sf
___
Nagios-users mailing list
Nagios-users@lists.sourceforge.net
https://lists.sourceforge.net/lists/listinfo/nagios-users
::: Please include Nagios version, plugin version (-v) and OS when reporting 
any issue. 
::: Messages without supporting info will risk being sent to /dev/null


Re: [Nagios-users] monitor http traffic

2011-03-31 Thread Jim Avery
On 31 March 2011 21:51, khurram aziz khurramazizkha...@yahoo.com wrote:
 Hi,

 I have a windows server 2003 based web server ( IIS) that has 5 websites
 running on it and each web site has a application pool allocated to it. What
 I want to do is to add this web server in my Nagios  so that I can keep an
 eye on whether these services are running or not and at the same time also
 to graph their performance using pnp4nagios...

 can some1 walk me through the configurations plz...


Wolfgang Barth walked me through all this sort of stuff.

http://nostarch.com/nagios.htm

hth,

Jim

--
Create and publish websites with WebMatrix
Use the most popular FREE web apps or write code yourself; 
WebMatrix provides all the features you need to develop and 
publish your website. http://p.sf.net/sfu/ms-webmatrix-sf
___
Nagios-users mailing list
Nagios-users@lists.sourceforge.net
https://lists.sourceforge.net/lists/listinfo/nagios-users
::: Please include Nagios version, plugin version (-v) and OS when reporting 
any issue. 
::: Messages without supporting info will risk being sent to /dev/null


Re: [Nagios-users] Nagios Weekly Service Check

2011-03-30 Thread Jim Avery
On 23 March 2011 09:39, Nair vman...@rediffmail.com wrote:

 Hi Friends

 I have below service check defined to perform check every Monday.

 define timeperiod{
 timeperiod_name only_mon
 alias Only on monday
 monday 11:00-17:00
 }

 define service{
 service_description TEST_SRV
 host_name localhost
 max_check_attempts 1
 normal_check_interval 120
 retry_check_interval 1
 check_period only_mon
 check_command TEST_CMD
 use template_TEST
 }
 The service was last reported on 2011-03-14 17:00:00.But,I am expected to get 
 check reuslt on 2011-03-21 11:00:00. Can you please give me an idea why the 
 weekly check schedule fails.

Firstly, note that Nagios will usually only record an entry in the log
if the status has changed.  Did you check the status of the service
and see what the last check time was?

If you only want one check at a specific time once a week, then
personally I would configure a script to run the check in cron and
send a passive check result to Nagios.It's maybe a bit of a pain
to learn how to do it the first time but a very useful technique for
all sorts of things.

hth,

Jim

--
Create and publish websites with WebMatrix
Use the most popular FREE web apps or write code yourself; 
WebMatrix provides all the features you need to develop and 
publish your website. http://p.sf.net/sfu/ms-webmatrix-sf
___
Nagios-users mailing list
Nagios-users@lists.sourceforge.net
https://lists.sourceforge.net/lists/listinfo/nagios-users
::: Please include Nagios version, plugin version (-v) and OS when reporting 
any issue. 
::: Messages without supporting info will risk being sent to /dev/null


Re: [Nagios-users] NSClient++. Monitoring the devices behind the Firewall.

2011-03-15 Thread Jim Avery
On 15 March 2011 13:47, C. Bensend be...@bennyvision.com wrote:

 If you're looking to do this without cooperation from the client
 and their security folks, you're going to run into problems.  If
 they want you to monitor their hosts, they have to provide some
 manner of accessing them.

 Just to be thorough, passive monitoring is also a possibility.
 In that case, each of the clients would be configured to send the
 service check results to the Nagios server, and would probably
 not require any changes to the firewall.

 However, I choose to use active monitoring, so I cannot help
 with that setup, nor would I necessarily recommend it.


I would echo what Benny said there having tried it myself.  Passive
monitoring is possible, and fairly easy to set up using the NSClient++
agent, but can be more trouble than it is worth as whenever you need
to make any change you have to log on to the remote server to change
the NSClient++ config.  If it's behind two firewalls then I would
guess that might not be easy.

If it's a large site with lots of servers, I would maybe consider
deploying a Nagios server on that site so there is only then one node
which needs to communicate back and forth with your central Nagios
server.  As ever, there are lots of different ways you can slice and
dice this problem.

--
Colocation vs. Managed Hosting
A question and answer guide to determining the best fit
for your organization - today and in the future.
http://p.sf.net/sfu/internap-sfd2d
___
Nagios-users mailing list
Nagios-users@lists.sourceforge.net
https://lists.sourceforge.net/lists/listinfo/nagios-users
::: Please include Nagios version, plugin version (-v) and OS when reporting 
any issue. 
::: Messages without supporting info will risk being sent to /dev/null


Re: [Nagios-users] NSClient++. Monitoring the devices behind theFirewall.

2011-03-15 Thread Jim Avery
On 15 March 2011 14:34, Daniel Wittenberg
daniel.wittenberg.r...@statefarm.com wrote:
 You could always have a passive check that calls home to get any new 
 updates so then you wouldn't really have to login to each one to push down 
 changes.

 Dan

That's a neat idea!

--
Colocation vs. Managed Hosting
A question and answer guide to determining the best fit
for your organization - today and in the future.
http://p.sf.net/sfu/internap-sfd2d
___
Nagios-users mailing list
Nagios-users@lists.sourceforge.net
https://lists.sourceforge.net/lists/listinfo/nagios-users
::: Please include Nagios version, plugin version (-v) and OS when reporting 
any issue. 
::: Messages without supporting info will risk being sent to /dev/null


Re: [Nagios-users] Acknowledgements with hard coded email address

2011-03-12 Thread Jim Avery
On 9 March 2011 18:05, Robert Bailey rob...@briworks.com wrote:
 I want to set Nagios so that no when you acknowledge either a host or service 
 problem with the Notify checkbox checked, the email goes to a specific 
 email address. Anyone know if this is possible? I've dug through the code a 
 little bit, but I'm not a programmer and I can't find any available 
 configuration options.

 Robert


Robert, I think you can configure a contact with the specific email
address, make it a contact (or member of the relevant contactgroup)
for the host or service, but most importantly in the contact
definition, have:

  host_notification_options n
or
  service_notification_options n

then the contact will still receive Acknowledgement notifications but
nothing else for that service or host.  Certainly this is what happens
if you specify n in the notification_options directive for a service
so I expect it's the same when you specify n in the contact
definition.  Give it a try anyway.

This is contrary to the documentation, so I wouldn't be too surprised
if it's 'fixed' in a future release, but if that's the case, I would
hope a a argument will be added to allow you to specify you want ack
notifications.  For the time being, give this a try.

hth,

Jim

--
Colocation vs. Managed Hosting
A question and answer guide to determining the best fit
for your organization - today and in the future.
http://p.sf.net/sfu/internap-sfd2d
___
Nagios-users mailing list
Nagios-users@lists.sourceforge.net
https://lists.sourceforge.net/lists/listinfo/nagios-users
::: Please include Nagios version, plugin version (-v) and OS when reporting 
any issue. 
::: Messages without supporting info will risk being sent to /dev/null


Re: [Nagios-users] NOC display screen

2011-02-27 Thread Jim Avery
On 27 February 2011 20:04, chiel ch...@gmx.net wrote:
 Hi all,

 I was wondering what everybody is using for there noc display screen. We
 have a display screen where I want to display both the host and service
 problems (if any). Using the normal nagios webpages we still have to
 click on either the service or host to display a particular problem.
 I see there is something called Naglite(v1,v2 and v3) which seems to do
 the trick. With a bit of tweaking this should work. Before I start
 configure I would like to know what you are all using.


A similar question came up here only a couple of weeks ago.  This was
my advice..

If it's any help to you, the URL I now use is:

http://webserver/nagios/cgi-bin/status.cgi?host=allhostprops=10servicestatustypes=29serviceprops=10sorttype=2sortoption=6noheader

Even though this doesn't explictly show hosts, if a host is down you
will still see it on this page because its services will also be down
(or unknown or whatever).

According to the Display Filters panel which appears if you remove the
noheader, this one shows:

Host Status Types:  All
Host Properties:Not In Scheduled Downtime  Has Not Been Acknowledged
Service Status Types:   Pending | Unknown | Warning | Critical
Service Properties: Not In Scheduled Downtime  Has Not Been Acknowledged

and the sort is by state duration descending.

Of course it won't show downed hosts if they have no services.  I make
sure all hosts have at least one service (even if that's just ping) so
they can be seen on this service page.

hth,

Jim

--
Free Software Download: Index, Search  Analyze Logs and other IT data in 
Real-Time with Splunk. Collect, index and harness all the fast moving IT data 
generated by your applications, servers and devices whether physical, virtual
or in the cloud. Deliver compliance at lower cost and gain new business 
insights. http://p.sf.net/sfu/splunk-dev2dev 
___
Nagios-users mailing list
Nagios-users@lists.sourceforge.net
https://lists.sourceforge.net/lists/listinfo/nagios-users
::: Please include Nagios version, plugin version (-v) and OS when reporting 
any issue. 
::: Messages without supporting info will risk being sent to /dev/null


  1   2   3   4   5   6   >